SlideShare a Scribd company logo
1 of 61
Download to read offline
27/04/2015 neetpgquiz.medicoapps.org/testCreation/preview.php?tid=18
http://neetpgquiz.medicoapps.org/testCreation/preview.php?tid=18 1/61
A CT scan of acute pancreatitis will show following features, except:
A: Poor contrast enhancement
B: Dilated main pancreatic duct
C: Enlargement of the pancreas
D: Ill defined outline of the pancreas
Correct Ans:B
Explanation
Dilatation of main pancreatic duct is usually seen in chronic calcific pancreatitis.
Ref: Albert L. Baert, Guy Delorme, L. Van Hoe (1999), Chapter 5 , “Pancreatic Disease in
The Childhood”, In the Book, “Radiology of The Pancreas”, Springer Publications, USA,
Page 110 ;  Text Book of Radiology and Imaging By Sutton, 7th Edition, Page 7790 
Sample Previous Year Question on Pancreatitis based on previous Year Questions of
NEET PG, USMLE,PLAB,FMGE (MCI Screening). Please visit www.medicoapps.org
for more such Quizzes
Colon cut off sign in plain X­Ray of abdomen is seen in?
A: Mesenteric ischemia
B: Intussusception
C: Acute pancreatitis
27/04/2015 neetpgquiz.medicoapps.org/testCreation/preview.php?tid=18
http://neetpgquiz.medicoapps.org/testCreation/preview.php?tid=18 2/61
D: Acute cholangitis
Correct Ans:C
Explanation
Abrupt termination of the gas within the colon at the level of the radiological splenic
flexure is called colon cut off sign. Inflammatory infiltration of the phrenicocolic ligament
leads to the spasm of the colon and lumen narrowing at the level where colon returns to
retroperitoneum.
Sample Previous Year Question on Pancreatitis based on previous Year Questions of
NEET PG, USMLE,PLAB,FMGE (MCI Screening). Please visit www.medicoapps.org
for more such Quizzes
“Strings of pearls” appearance is seen in?
A: SAIO
B: Gallstone ileus
C: Chronic pancreatitis
D: Duodenal perforation
Correct Ans:C
Explanation
Chronic pancreatitis is characterized by irregularities of of the pancreatic ducts, ductal
strictures, and area of duct dilation. The major as well as the side branch ducts may be
involved. For unexplained reason, some patients with chronic pancreatitis develop dilated
main pancreatic ducts (large ductal disease), whereas others retain ducts of normal or
even smaller than normal calibers (small ductal disease). Some patients with chronic
pancreatitis can be shown to have major ducts that have the appearance of a “chain of
lakes” or a ”strings of pearls” that is the result of segment of dilated duct separated by
areas of ductal stricture.
Ref: Sutton’s Radiology 7/e, Volume 1, Page 798
27/04/2015 neetpgquiz.medicoapps.org/testCreation/preview.php?tid=18
http://neetpgquiz.medicoapps.org/testCreation/preview.php?tid=18 3/61
Sample Previous Year Question on Pancreatitis based on previous Year Questions of
NEET PG, USMLE,PLAB,FMGE (MCI Screening). Please visit www.medicoapps.org
for more such Quizzes
On radiography widened duodenal 'C' loop with irregular mucosal pattern on upper
gastrointestinal barium series is most likely due to:
A: Duodenal ileus
B: Chronic pancreatitis
C: Duodenal ulcer
D: Carcinoma head of pancreas
Correct Ans:D
Explanation
Widening of duodenal C loop, inverted 3 sign of Frostburg and rose
thorn appearance is seen in carcinoma head of pancreas. The
treatment is Whipple’s procedure.
Ref: Albert L. Baert, Guy Delorme, L. Van Hoe (1999), Chapter 5, “
Pancreatic Disease in The Childhood”, In the Book, “Radiology of
The Pancreas”, Springer Publications, USA, Page 116
Sample Previous Year Question on Pancreatitis based on previous Year Questions of
NEET PG, USMLE,PLAB,FMGE (MCI Screening). Please visit www.medicoapps.org
for more such Quizzes
A CT scan of acute pancreatitis will show following features, except:
A: Poor contrast enhancement
B: Dilated main pancreatic duct
27/04/2015 neetpgquiz.medicoapps.org/testCreation/preview.php?tid=18
http://neetpgquiz.medicoapps.org/testCreation/preview.php?tid=18 4/61
C: Enlargement of the pancreas
D: Ill defined outline of the pancreas
Correct Ans:B
Explanation
Dilatation of main pancreatic duct is usually seen in chronic calcific pancreatitis.
Ref: Albert L. Baert, Guy Delorme, L. Van Hoe (1999), Chapter 5 , “Pancreatic Disease in
The Childhood”, In the Book, “Radiology of The Pancreas”, Springer Publications, USA,
Page 110 ;  Text Book of Radiology and Imaging By Sutton, 7th Edition, Page 7790 
Sample Previous Year Question on Pancreatitis based on previous Year Questions of
NEET PG, USMLE,PLAB,FMGE (MCI Screening). Please visit www.medicoapps.org
for more such Quizzes
The primary imaging technique used in a case of suspected acute pancreatitis is:
A: MRI abdomen
B: CECT abdomen with pancreatic protocol
C: USG abdomen
D: MR enterography
Correct Ans:B
Explanation
MDCT is the primary imaging technique used in evaluating patients suspected of having
acute pancreatitis.
CT useful for:
Morphologic changes in the pancreas that allow confirmation of pancreatitis 
Assessment of the severity  
27/04/2015 neetpgquiz.medicoapps.org/testCreation/preview.php?tid=18
http://neetpgquiz.medicoapps.org/testCreation/preview.php?tid=18 5/61
It is the only imaging study that predicts clinical outcome.
Ref: ACR Appropriateness Criteria, Acute Pancreatitis
 
Sample Previous Year Question on Pancreatitis based on previous Year Questions of
NEET PG, USMLE,PLAB,FMGE (MCI Screening). Please visit www.medicoapps.org
for more such Quizzes
The CT severity index in acute pancreatitis is described by:
A: Balthazar
B: Mengini
C: Chapman
D: Napelon
Correct Ans:A
Explanation
The CT severity index (CTSI)­described by Balthazar
Balthazar CT Severity Index
      CT Grade           Score
A. Normal­     0
B. Enlarged gland ­1
C. Peri­pancreatic inflammation­ 2
D. One fluid collection­ 3
E. Two or more collections­ 4
Necrosis Score
<30% ­(2)
30%­50%­ (4)
>50% ­(6)
Ref: ACR Appropriateness Criteria,Acute Pancreatitis.
27/04/2015 neetpgquiz.medicoapps.org/testCreation/preview.php?tid=18
http://neetpgquiz.medicoapps.org/testCreation/preview.php?tid=18 6/61
Sample Previous Year Question on Pancreatitis based on previous Year Questions of
NEET PG, USMLE,PLAB,FMGE (MCI Screening). Please visit www.medicoapps.org
for more such Quizzes
The potential limitation of CECT in the setting of the acute pancreatitis is:
A: Can not assess the severity
B: May not useful in assessing prognosis
C: Can not clearly diagnose gallstone pancreatitis
D: Can not diagnose peripancreatic fluid collection
Correct Ans:C
Explanation
MDCT is the primary imaging technique used in evaluating patients suspected of having
acute pancreatitis
CT useful for:
Morphologic changes in the pancreas that allow confirmation of pancreatitis 
Assessment of the severity  
It is the only imaging study that predicts clinical outcome.
A potential limitation of MDCT in assessing acute pancreatitis is that it has only moderate
sensitivity for
detecting gallstones and biliary stones. 
Ref: ACR Appropriateness Criteria,Acute Pancreatitis.
 
Sample Previous Year Question on Pancreatitis based on previous Year Questions of
NEET PG, USMLE,PLAB,FMGE (MCI Screening). Please visit www.medicoapps.org
for more such Quizzes
A patient with abdominal pain shows "coffee bean" sign in plain abdominal X­ray. He
probably have:
27/04/2015 neetpgquiz.medicoapps.org/testCreation/preview.php?tid=18
http://neetpgquiz.medicoapps.org/testCreation/preview.php?tid=18 7/61
A: Colon carcinoma
B: Duodenal atresia
C: Acute pancreatitis
D: Sigmoid volvulus
Correct Ans:D
Explanation
Sigmoid volvulus is a closed­loop colonic obstruction due to twisting along the mesenteric
or long axis of the bowel. Although colonic volvulus is not common, about 90% of cases
occur in the sigmoid colon. On plain abdominal films, the sigmoid volvulus forms an
inverted U­shaped structure with the twisted sigmoid loops lying adjacent and having an
oval appearance called the "coffee bean" sign. On barium enema examination, tapered
obstruction of the sigmoid colon is found.
                          
Ref: Ott D.J. (2011). Chapter 10. Gastrointestinal Tract. In M.Y. Chen, T.L. Pope, D.J. Ott
(Eds), Basic Radiology, 2e.
Sample Previous Year Question on Pancreatitis based on previous Year Questions of
NEET PG, USMLE,PLAB,FMGE (MCI Screening). Please visit www.medicoapps.org
for more such Quizzes
Double­bubble sign on abdominal X­ray is seen in :
A: Colon carcinoma
B: Duodenal atresia
C: Acute pancreatitis
D: Perforation
27/04/2015 neetpgquiz.medicoapps.org/testCreation/preview.php?tid=18
http://neetpgquiz.medicoapps.org/testCreation/preview.php?tid=18 8/61
Correct Ans:B
Explanation
Double bubble sign is the characteristic abdominal Xray sign associated with duodenal
atresia.  In tis, the stomach and first part of the duodenum are the only parts of the
intestine filled with gas giving the characteristic appearance. 
 
Duodenal atresia typically presents in the first hour of life. Children presents with bilious
vomiting and epigastric distention within a few hours of birth. Meconium may be passed
normally.
 
It is often associated with other conditions such as esophageal atresia, intestinal atresia,
cardiac and renal anomalies. 
 
Ref: Oski's Pediatric Certification and Recertification Board Review  By Carmen Coombs,
page 404. Sundaram S.S., Hoffenberg E.J., Kramer R.E., Sondheimer J.M., Furuta G.T.
(2012). Chapter 21. Gastrointestinal Tract. In W.W. Hay, Jr., M.J. Levin, R.R. Deterding,
J.J. Ross, J.M. Sondheimer (Eds), CURRENT Diagnosis & Treatment: Pediatrics, 21e.
Sample Previous Year Question on Pancreatitis based on previous Year Questions of
NEET PG, USMLE,PLAB,FMGE (MCI Screening). Please visit www.medicoapps.org
for more such Quizzes
Erythema nodosum is seen in all of the following, EXCEPT:
A: Pregnancy
B: Tuberculosis
C: SLE
D: Chronic Pancreatitis
Correct Ans:D
Explanation
Pregnancy, SLE and Tuberculosis are all associated with Erythema Nodosum. Erythema Nodosum are
painfully red nodular lesions, occuring bilaterally on the shins. This lesion occur predominantly in
woman in the fall and winter months. It is associated with SLE, Leprosy, TB, etc.
Ref: Immunology, Immunopathology and Immunity, by Stewart Sell, Edward E. Max, Page
518; Chronic Pancreatitis, by Markus Buchler, Helmutt Friess, Page 29, 30, 37.
27/04/2015 neetpgquiz.medicoapps.org/testCreation/preview.php?tid=18
http://neetpgquiz.medicoapps.org/testCreation/preview.php?tid=18 9/61
Sample Previous Year Question on Pancreatitis based on previous Year Questions of
NEET PG, USMLE,PLAB,FMGE (MCI Screening). Please visit www.medicoapps.org
for more such Quizzes
Pancreatitis occurs with:
A: Abacavir
B: Zidovudine
C: Lamivudine
D: Didanosine
Correct Ans:D
Explanation
The most prominent dose related toxicity associated with the use of didanosine are
pancreatitis and peripheral neuropathy.
Ref: Essentials of Medical Pharmacology By KD Tripathi, 5th Edition, Page 728 ­ 30;
Pharmacogenomic Testing in Current Clinical Practice : Implementation in the Clinical
Laboratory By Alan H. B. Wu, Kiang­Teck J. Yeo, Page 203.
Sample Previous Year Question on Pancreatitis based on previous Year Questions of
NEET PG, USMLE,PLAB,FMGE (MCI Screening). Please visit www.medicoapps.org
for more such Quizzes
All of the following drugs cause pancreatitis as side effect, EXCEPT:
A: Captopril
B: Furosemide
C: Valproic acid
27/04/2015 neetpgquiz.medicoapps.org/testCreation/preview.php?tid=18
http://neetpgquiz.medicoapps.org/testCreation/preview.php?tid=18 10/61
D: Didanosine
Correct Ans:A
Explanation
Adverse effects of captopril are hypotension, acute renal failure in patients with bilateral
renal artery stenosis, hyperkalemia , dry cough and angioedema.
Drugs associated with highest incidence of pancreatitis are
didanosine, sodium valproate, 5 aminosalicylates and oestrogens.
Drugs causing pancreatitis:
Didanosine
Furosemide and thiazide diuretics
Mesalazine
Metronidazole
Oestrogens
Pentamidine
Sodium valproate
Sulfasalazine
Sulfonamides
Tetracycline
Ref: Adverse Drug Reactions  edited by Anne Lee page 183.
Benowitz N.L. (2012). Chapter 11. Antihypertensive Agents. In B.G.
Katzung, S.B. Masters, A.J. Trevor (Eds), Basic & Clinical
Pharmacology, 12e.
Sample Previous Year Question on Pancreatitis based on previous Year Questions of
NEET PG, USMLE,PLAB,FMGE (MCI Screening). Please visit www.medicoapps.org
for more such Quizzes
The MOST important risk factor for pancreatic cancer among the following is:
A: Cigarette smoking
B: Chronic pancreatitis
C: Diabetes
27/04/2015 neetpgquiz.medicoapps.org/testCreation/preview.php?tid=18
http://neetpgquiz.medicoapps.org/testCreation/preview.php?tid=18 11/61
D: Alcohol
Correct Ans:A
Explanation
All are risk factors of pancreatic malignancy except alcohol.
Cigarette smoking is the most common environmental risk factor for pancreatic
malignancy. Other risk factors are chronic pancreatitis and diabetes. 
ALSO NOTE:
Alcohol does not appear to be a risk factor.
Ref: Harrison, Edition ­18, Page­787
Sample Previous Year Question on Pancreatitis based on previous Year Questions of
NEET PG, USMLE,PLAB,FMGE (MCI Screening). Please visit www.medicoapps.org
for more such Quizzes
A 25 year old plumber comes to the clinic with complaints of abdominal colic,
constipation, weakness of hand and anemia since 2 years. What would be the most
probable diagnosis in this patient?
A: Lead poisoning
B: Gastric carcinoma
C: Chronic pancreatitis
D: Hookworm infestation
Correct Ans:A
Explanation
The occupational history and clinical features of this patient suggest that he is suffering
from chronic lead poisoning. 
Characteristic features of chronic lead poisoning includes:
27/04/2015 neetpgquiz.medicoapps.org/testCreation/preview.php?tid=18
http://neetpgquiz.medicoapps.org/testCreation/preview.php?tid=18 12/61
Facial pallor: Particularly of the mouth is the earliest and most consistent sign.
Anemia: Anemia associated with polychromasia, punctate basophilia,
reticulocytosis, poikolocytosis, anisocytosis and sideroblastosis. Basophilic stippling
refers to the presence of dark blue pin head sized spots in the cytoplasm of red
blood cells. 
Lead line or Burtonian line: seen on gums in 70% cases.
Colic and constipation seen in 85% cases.
Lead palsy: Radial nerve is most commonly involved resuling in wrist drop. 
Encephalopathy: It is seen in every case of plumbism. Lead encephalopathy is
irreversible and 85% have permanent brain damage.
Sample Previous Year Question on Pancreatitis based on previous Year Questions of
NEET PG, USMLE,PLAB,FMGE (MCI Screening). Please visit www.medicoapps.org
for more such Quizzes
Round worm causes all, except :
A: Obstructive jaundice
B: Pancreatitis
C: Intestinal obstruction
D: Intestinal perforation
Correct Ans:D
Explanation
A large Round worm burden can lead to malnutrition and weakness, and a mass of worms
may lead to bowel obstruction. Wandering ascaris also traverse internal organs rarely
leading to biliary obstruction, hepatic abscess, acute pancreatitis, acute appendicitis, or
hypersensitivity pneumonitis. It does not cause intestinal perforation.
 
Ref :VanRooyen M.J., Venugopal R. (2011). Chapter 156. World Travelers. In R.K.
Cydulka, G.D. Meckler (Eds), Tintinalli's Emergency Medicine: A Comprehensive Study
Guide, 7e.
Sample Previous Year Question on Pancreatitis based on previous Year Questions of
NEET PG, USMLE,PLAB,FMGE (MCI Screening). Please visit www.medicoapps.org
for more such Quizzes
27/04/2015 neetpgquiz.medicoapps.org/testCreation/preview.php?tid=18
http://neetpgquiz.medicoapps.org/testCreation/preview.php?tid=18 13/61
Which of the following condition is associated with migratory thrombophlebitis?
A: Pancreatitis
B: Pancreatic Ca
C: Varicose veins
D: Buerger's disease
Correct Ans:C
Explanation
Superficial vein thrombophlebitis (SVT) most commonly occurs in varicose veins but can
occur in normal veins. When SVT recurs at variable sites in normal superficial veins, it
may signify a hidden visceral malignancy or a systemic disease such as a blood dyscrasia
and/or a collagen vascular disease. This condition is known as thrombophlebitis migrans.
Clinical signs of SVT include redness, warmth, and tenderness along the distribution of the
affected veins, often associated with a palpable cord.
 
Ref: Liem T.K., Moneta G.L. (2010). Chapter 24. Venous and Lymphatic Disease. In F.C.
Brunicardi, D.K. Andersen, T.R. Billiar, D.L. Dunn, J.G. Hunter, J.B. Matthews, R.E. Pollock
(Eds), Schwartz's Principles of Surgery, 9e.
Sample Previous Year Question on Pancreatitis based on previous Year Questions of
NEET PG, USMLE,PLAB,FMGE (MCI Screening). Please visit www.medicoapps.org
for more such Quizzes
All of the following statements about Pancreatic Carcinoma are true, EXCEPT:
A: Mutation in P53 gene is associated in 75% of cases
B: Hereditary Pancreatitis significantly increases the risk
C: Median survival in locally advanced (stage III) disease is 3­6 months
27/04/2015 neetpgquiz.medicoapps.org/testCreation/preview.php?tid=18
http://neetpgquiz.medicoapps.org/testCreation/preview.php?tid=18 14/61
D: Five year survival after curative pancreaticoduodenectomy is 15 ­20%
Correct Ans:C
Explanation
Median survival for stage III pancreatic cancer is 6­10 months and for patients with
metastatic disease such as stage IV it is 3­6 months.
Staging of pancreatic cancer:
Stage I disease includes T1 (size <2cm diameter) and T2 tumors (>2cm, limited to
pancreas) with no lymph node involvement. 
Stage II disease includes T3 ( lesions extend beyond the pancreas but do not involve the
celiac axis or superior mesenteric artery). 
Stage III includes T4 (lesions involve the celiac axis or superior mesenteric artery and
are not resectable) without metastatic disease. 
Stage IV includes T4 with metastases to distant sites such as the liver or lungs are stage
IV.
Ref: Schwartz's Principles of Surgery, 9th Edition, Chapter 33; Pancreatic Cancer By
Daniel D. Von Hoff, Page 447; Lecture Notes: General Surgery By Harold Ellis, Page 288
Sample Previous Year Question on Pancreatitis based on previous Year Questions of
NEET PG, USMLE,PLAB,FMGE (MCI Screening). Please visit www.medicoapps.org
for more such Quizzes
A patient presents in the causality with history of abdominal pain. On examination the
person is in shock, with severe abdominal tenderness and guarding. There was also one
episode of bloody diarrhoea. He gives a history of recurrent abdominal pain soon after
taking food which persists for about 3 hours after food. He also has a history of MI about 5
years back. What is your diagnosis?
A: Acute thrombotic mesenteric vascular occlusion
B: Acute pancreatitis
C: Acute duodenal ulcer perforation
D: Acute appendicitis
27/04/2015 neetpgquiz.medicoapps.org/testCreation/preview.php?tid=18
http://neetpgquiz.medicoapps.org/testCreation/preview.php?tid=18 15/61
Correct Ans:A
Explanation
The person has had recurrent episodes of abdominal angina as shown by the pain after
food intake. He also has a history of MI which also suggests that he has atherosclerosis.
The bloody diarrhoea is again suggestive of mesenteric vascular occlusion in this patient.
 
Ref:  Manipal Millennium Edition, Page 429; S.Das, Edition 5, Page 354
Sample Previous Year Question on Pancreatitis based on previous Year Questions of
NEET PG, USMLE,PLAB,FMGE (MCI Screening). Please visit www.medicoapps.org
for more such Quizzes
Which of the following type of pancreatitis is associated with better prognosis ?
A: Alcoholic pancreatitis
B: Gall stone pancreatitis
C: Post operative pancreatitis
D: Idiopathic pancreatitis
Correct Ans:B
Explanation
Pancreatitis caused by a remediable cause such as cholilithiasis is associated with best prognosis. 
Eradication of gallsones can prevent further attacks of pancreatitis.
Gall stone pancreatitis accounts for about 90% of cases of acute pancreatitis. Women are more 
affected than men. In this, gallstone is typically located in the distal common bile duct, which shares 
its pathway with the main pancreatic duct. Gallstones are recoverable in the feces of over 90% of 
patients within 10days of an attack of acute pancreatitis.
Treatment involves supportive care till amylase and lipase levels returns to normal. ERCP with 
sphincterectomy and stone retrieval is done in patients who do not pass the stone spontaneously. 
After the resolution of of an episode of gallstone pancreatitis, patient should be treated with a 
laparoscopic hysterectomy with a cholangiogram.
Ref: Oxford Textbook of Surgery, 2nd Edition, Page 1766; Clinical Review of Surgery: ABSITE 
27/04/2015 neetpgquiz.medicoapps.org/testCreation/preview.php?tid=18
http://neetpgquiz.medicoapps.org/testCreation/preview.php?tid=18 16/61
Preparation By Surgisphere Corporation, Page 237­8
Sample Previous Year Question on Pancreatitis based on previous Year Questions of
NEET PG, USMLE,PLAB,FMGE (MCI Screening). Please visit www.medicoapps.org
for more such Quizzes
A patient undergoes a prolonged and complicated pancreatic surgery for chronic
pancreatitis. Most preferred route for supplementary nutrition in this patient would be:
A: Total Parental Nutrition
B: Feeding Gastrostomy
C: Feeding Jejunostomy
D: Oral feeding
Correct Ans:C
Explanation
When a patient has undergone a prolonged and complicated pancreatic surgery, the
recovery of the patient will require a form of nutrition that gives rest to the pancreas and
protect the pancreatic anastomosis. A feeding jejunostomy in such a case can provide
supplementary nutrition and also gives protection to the pancreatic anastomosis and
cause minimal stimulation of pancreatic secretion thereby giving rest to the pancreas.
 
Ref: Clinical Nutrition in Gastrointestinal Disease By Buchman, 2006, Pages 256­57;
Artificial Nutrition Support in Clinical Practice By Jason Payne, 2nd Edition, Page 273;
Essentials of General Surgery By Lawrence, 4th Edition, Page 80
Sample Previous Year Question on Pancreatitis based on previous Year Questions of
NEET PG, USMLE,PLAB,FMGE (MCI Screening). Please visit www.medicoapps.org
for more such Quizzes
All of the following statements about Pancreatic Carcinoma are true, EXCEPT:
27/04/2015 neetpgquiz.medicoapps.org/testCreation/preview.php?tid=18
http://neetpgquiz.medicoapps.org/testCreation/preview.php?tid=18 17/61
A: Mutation in P53 gene is associated in 75% of cases
B: Hereditary Pancreatitis significantly increases the risk
C: Median survival in locally advanced (stage III) disease is 3­6 months
D: Five year survival after curative pancreaticoduodenectomy is 15 ­20%
Correct Ans:C
Explanation
Median survival for stage III pancreatic cancer is 6­10 months and for patients with
metastatic disease such as stage IV it is 3­6 months.
Staging of pancreatic cancer:
Stage I disease includes T1 (size <2cm diameter) and T2 tumors (>2cm, limited to
pancreas) with no lymph node involvement. 
Stage II disease includes T3 ( lesions extend beyond the pancreas but do not involve the
celiac axis or superior mesenteric artery). 
Stage III includes T4 (lesions involve the celiac axis or superior mesenteric artery and
are not resectable) without metastatic disease. 
Stage IV includes T4 with metastases to distant sites such as the liver or lungs are stage
IV.
Ref: Schwartz's Principles of Surgery, 9th Edition, Chapter 33; Pancreatic Cancer By
Daniel D. Von Hoff, Page 447; Lecture Notes: General Surgery By Harold Ellis, Page 288
Sample Previous Year Question on Pancreatitis based on previous Year Questions of
NEET PG, USMLE,PLAB,FMGE (MCI Screening). Please visit www.medicoapps.org
for more such Quizzes
A lady with a history of epigastric pain radiating to back for three days with normal Serum
amylase  levels  and  abdomen  ultrasonogram  (USG)  revealed  cholelithiasis  and  an
enlarged pancreas. CT scan confirmed the underlying pathology. Which of the following is
the most likely diagnosis.
A: Acute Cholecystitis
B: Acute Pancreatitis
27/04/2015 neetpgquiz.medicoapps.org/testCreation/preview.php?tid=18
http://neetpgquiz.medicoapps.org/testCreation/preview.php?tid=18 18/61
C: Acute Appendicitis
D: Acute Peritonitis
Correct Ans:B
Explanation
History of epigastric pain radiating to back along with an enlarged (edematous) pancreas
on ultrasonography suggests a diagnosis of Acute Pancreatitis. Serum amylase levels may
be normal during an attack of acute pancreatitis.
 
Ref: Bailey and Love Short Practice of Surgery 24th Edition, Page1123­1125; Harrison's
17th Edition, Page 2003, 2004, 2007; Current Diagnosis & Treatment in Gastroenterology
(Wiley Blackwell) 4th Edition, Page 516; Basic skills in Interpreting Laboratory Data by
Mary Lee 4th Edition, Page 7251.
Sample Previous Year Question on Pancreatitis based on previous Year Questions of
NEET PG, USMLE,PLAB,FMGE (MCI Screening). Please visit www.medicoapps.org
for more such Quizzes
Which of the following can be used in the management of acute pancreatitis?
A: Octreotide
B: Aprotinin
C: Glucagone
D: All the above
Correct Ans:D
Explanation
Antibiotic, corticosteroids, H2
 blockers and NSAIDS also have a role.
Sample Previous Year Question on Pancreatitis based on previous Year Questions of
NEET PG, USMLE,PLAB,FMGE (MCI Screening). Please visit www.medicoapps.org
for more such Quizzes
27/04/2015 neetpgquiz.medicoapps.org/testCreation/preview.php?tid=18
http://neetpgquiz.medicoapps.org/testCreation/preview.php?tid=18 19/61
Which of the following is not a cause for Acute pancreatitis?
A: Gall stones
B: Hemochromatosis
C: Alcohol intake
D: Anti retroviral drug
Correct Ans:B
Explanation
Common causes of acute pancreatitis are gallstones, alcohol, ERCP, trauma, post surgery, dysfunction 
of sphincter of Oddi and by drugs such as antiretroviral drugs, azathioprine, tetracycline, valproic acid, 
6 mercaptopurine, sulfonamides and estrogen. Hemochromatosis is not a cause of acute pancreatitis.
Reference:
Harrison's Principles of Internal Medicine 18e, chapter 313.
Sample Previous Year Question on Pancreatitis based on previous Year Questions of
NEET PG, USMLE,PLAB,FMGE (MCI Screening). Please visit www.medicoapps.org
for more such Quizzes
All of the following are used in the treatment of acute pancreatitis EXCEPT:
A: Analgesics
B: IV fluids
C: Antibiotics
D: Nasojejunal feeds
27/04/2015 neetpgquiz.medicoapps.org/testCreation/preview.php?tid=18
http://neetpgquiz.medicoapps.org/testCreation/preview.php?tid=18 20/61
Correct Ans:C
Explanation
In most patients (85­90%) with acute pancreatitis, the disease is self­limited and subsides 
spontaneously, usually within three to seven days after treatment is instituted. Conventional 
measures includes
•Analgesics for pain
•IV fluids and colloids to maintain normal intravascular volume
•No oral alimentation.
Once  it  is  clear  that  a  patient  will  not  be  able  to  tolerate  oral  feeding  (a  determination  that  can 
usually  be  made  within  48­72  hours),  enteral  nutrition  should  be  considered  [rather  than  total 
parenteral  nutrition  (TPN)]  since  it  maintains  gut  barrier  integrity,  thereby  preventing  bacterial 
translocation, is less expensive, and has fewer complications than TPN.
 
The route through which enteral feeding is administered is under debate. Nasogastric access is easier 
to  establish  and  may  be  as  safe  as  nasojejunal  enteral  nutrition.  However,  enteral  nutrition  that 
bypasses  the  stomach  and  duodenum  stimulates  pancreatic  secretions  less  and  this  rationale 
theoretically supports the use of the nasojejunal route. It has not been demonstrated whether either 
route  is  superior  in  altering  morbidity  and  mortality.  When  patients  with  necrotizing  pancreatitis 
begin  oral  intake  of  food,  consideration  should  also  be  given  to  the  addition  of  pancreatic enzyme
supplementation and proton pump inhibitor therapy to assist with fat digestion and reduce gastric 
acid.
About oral antibiotics harrisson states that:
"There is currently no role for prophylactic antibiotics in either interstitial or necrotizing pancreatitis. 
Although several early studies suggested a role for prophylactic antibiotics in patients with necrotizing 
pancreatitis, two recent double­blind, randomized controlled trials failed to demonstrate a reduction in 
pancreatic infection with use of antibiotic prophylaxis".
 
Percutaneous aspiration of necrosis with Gram stain and culture should generally not be performed 
until at least 7­10 days after establishing a diagnosis of necrotizing pancreatitis and only if there are 
ongoing signs of possible pancreatic infection such as sustained leukocytosis, fever, or organ failure. 
Once a diagnosis of infected necrosis is established, appropriate antibiotics should be instituted and 
surgical debridement should be undertaken
Sample Previous Year Question on Pancreatitis based on previous Year Questions of
NEET PG, USMLE,PLAB,FMGE (MCI Screening). Please visit www.medicoapps.org
for more such Quizzes
Which of the following statement regarding pancreatitis is true ?
A: Acute pancreatitis has reversible changes
27/04/2015 neetpgquiz.medicoapps.org/testCreation/preview.php?tid=18
http://neetpgquiz.medicoapps.org/testCreation/preview.php?tid=18 21/61
B: Alcohol causes only acute pancreatitis
C: Chronic pancreatitis shows no signs of inflammation
D: Acute pancreatitis affects mainly younger population
Correct Ans:A
Explanation
Chronic pancreatitis is a disease process characterized by irreversible damage to the pancreas as 
distinct from the reversible changes noted in acute pancreatitis. The condition is best defined by the 
presence of histologic abnormalities, including chronic inflammation, fibrosis, and progressive 
destruction of both exocrine and eventually endocrine tissue.
Alcohol is implicated in both acute and chronic pancreatitis and both can occur in young adults.
Sample Previous Year Question on Pancreatitis based on previous Year Questions of
NEET PG, USMLE,PLAB,FMGE (MCI Screening). Please visit www.medicoapps.org
for more such Quizzes
Scenario: A 40 year old immunocompromised lady presents with rapid onset of severe
addominal symptoms. 
 
Assertion: “Sentinel loop” is a X­ray finding in acute pancreatitis.
 
Reason: The finding is due to patient's posture of leaning forward like a loop, because of pain.
A:
Both Assertion and Reason are true, and Reason is the correct explanation for
Assertion
B:
Both Assertion and Reason are true, and Reason is not the correct explanation
for Assertion
C: Assertion is true, but Reason is false
D: Assertion is false, but Reason is true
27/04/2015 neetpgquiz.medicoapps.org/testCreation/preview.php?tid=18
http://neetpgquiz.medicoapps.org/testCreation/preview.php?tid=18 22/61
Correct Ans:C
Explanation
It is an X­ray finding of acute pancreatitis. Sentinel loop is a single dilated atonic loop of
small bowel in straight X­ray which is a diagnostic feature of acute pancreatitis.
Ref: A Manual of Clinical Surgery by S. Das, 6th Edition, Page 352
Sample Previous Year Question on Pancreatitis based on previous Year Questions of
NEET PG, USMLE,PLAB,FMGE (MCI Screening). Please visit www.medicoapps.org
for more such Quizzes
The 'beading' appearance is seen in the condition:
A: Chronic pancreatitis
B: Acute pancreatitis
C: Gall stone ileus
D: Sub­acute intestinal obstruction
Correct Ans:A
Explanation
In chronic pancreatitis, the pancreatic duct may show beading or a chain­of­lakes or
string­of­pearls appearance because of alternating stenosis and dilatation of the
pancreatic duct. Other features of chronic pancreatitis include strictures, cysts and ductal
calculi.
Sample Previous Year Question on Pancreatitis based on previous Year Questions of
NEET PG, USMLE,PLAB,FMGE (MCI Screening). Please visit www.medicoapps.org
for more such Quizzes
Pancreatitis may be produced by which of the following drugs?
27/04/2015 neetpgquiz.medicoapps.org/testCreation/preview.php?tid=18
http://neetpgquiz.medicoapps.org/testCreation/preview.php?tid=18 23/61
A: Colchicine
B: L­ asparagine
C: Ciprofloxacin
D: Nalidixic acid
Correct Ans:B
Explanation
Exposure to certain drugs is the third most common frequent cause of pancreatitis after
gallstones and alcohol.
Drugs associated with pancreatitis:
Definitive cause
5­ Aminosalicylate
6­ Mercaptopurine
Azathioprine
Cytosine arabinoside
Dideoxyinosine
Diuretics
Estrogens
Furosemide
Metronidazole
Pentamidine
Tetracycline
Thiazide
Trimethoprim ­ sulphamethoxazole
Valproic acid
Probable cause
Acetaminophen
a­ Methyl­ DOPA
Isoniazid
L­ asparagine
Phenformin
Procainamide
Sulindac
 
Ref: Sabiston 18/e, Page 1595
27/04/2015 neetpgquiz.medicoapps.org/testCreation/preview.php?tid=18
http://neetpgquiz.medicoapps.org/testCreation/preview.php?tid=18 24/61
Sample Previous Year Question on Pancreatitis based on previous Year Questions of
NEET PG, USMLE,PLAB,FMGE (MCI Screening). Please visit www.medicoapps.org
for more such Quizzes
Which of the following is NOT a bad prognostic sign for pancreatitis?
A: TLC > 16000
B: Calcium < 8 mmol/L
C: Glucose > 200 mg%
D: Prothrombin > 2 time the control
Correct Ans:D
Explanation
Ranson’s Prognostic signs of Acute Pancreatitis (Caused by causes other than gallstones)
At Admission  During the initial 48 hrs
 Age > 55 years
 WBC > 16,000/mm^3
 Blood glucose > 200 mg/dl
 Serum LDH > 350 IU/dl
 Serum AST > 250 U/dl
 Hematocrit fall > 10 points
 BUN elevation > 5 mg/dL
 Serum calcium < 8 mg/dL
 Arterial PO2 < 60 mmHg
 Base deficit > 4 mEq/dL
 Estimated fluid sequestration > 6L
 Also know
 Prognosis criteria for gallstone pancreatitis
At Admission During the initial 48 hrs
 Age > 70 years
 WBC > 18,000/mm^3
 Blood glucose > 220 mg/dl
 Serum LDH > 400 IU/dl
 Serum AST > 250 U/dl
 Hematocrit fall > 10 points
 BUN elevation > 2 mg/dL
 Serum calcium < 8 mg/dL
 Arterial PO2 < 60 mmHg
 Base deficit > 5 mEq/dL
 Estimated fluid sequestration > 4L
 
Ref: Sabiston 18/e, Page 1600; Schwartz 9/e, Page 1183; Washington Manual of Surgery
4/e, Page 282v.
27/04/2015 neetpgquiz.medicoapps.org/testCreation/preview.php?tid=18
http://neetpgquiz.medicoapps.org/testCreation/preview.php?tid=18 25/61
Sample Previous Year Question on Pancreatitis based on previous Year Questions of
NEET PG, USMLE,PLAB,FMGE (MCI Screening). Please visit www.medicoapps.org
for more such Quizzes
Poor prognostic factors for acute pancreatitis are all, EXCEPT:
A: Hyperglycemia
B: Hypocalcemia
C: Raised LDH levels in blood
D: Hyperamylasemia
Correct Ans:D
Explanation
Hyperamylasemia is not mentioned as the poor prognostic factor. Increased blood glucose,
Increased LDH, and decreased serum calcium, all are included in the ranson’s criteria of
severe pancreatitis.
Ref: Sabiston 18/e, Page 1600; Schwartz 9/e, Page 1183; Washington Manual of Surgery
4/e, Page 282.
Sample Previous Year Question on Pancreatitis based on previous Year Questions of
NEET PG, USMLE,PLAB,FMGE (MCI Screening). Please visit www.medicoapps.org
for more such Quizzes
All of the following can be used to predict severe acute pancreatitis, EXCEPT:
A: Glasgow score 3
27/04/2015 neetpgquiz.medicoapps.org/testCreation/preview.php?tid=18
http://neetpgquiz.medicoapps.org/testCreation/preview.php?tid=18 26/61
B: Apache II score 9
C: CT severity score 6
D: C reactive protein < 100
Correct Ans:D
Explanation
On account of difference in the outcome between patients with mild and severe disease, it
is important to define that group of patients who will develop severe pancreatitis.
Criteria used to determine the severity of acute pancreatitis:
 
Criteria used Score at which severity is indicated
Apache score 8
Glasgow score 3
Ranson criteria 3
CT severity index 6
C­ reactive protein > 150 mg L
Ref: Harrison 17/e, Page 2008; Bailey & Love 1140.
Sample Previous Year Question on Pancreatitis based on previous Year Questions of
NEET PG, USMLE,PLAB,FMGE (MCI Screening). Please visit www.medicoapps.org
for more such Quizzes
Persistent organ failure in the setting of acute pancreatitis is defined, when the organ
failure lasts for more than:
A: 12 hours
B: 24 hours
27/04/2015 neetpgquiz.medicoapps.org/testCreation/preview.php?tid=18
http://neetpgquiz.medicoapps.org/testCreation/preview.php?tid=18 27/61
C: 48 hours
D: 72 hours
Correct Ans:C
Explanation
The determinant of the severity of acute pancreatitis during the early phase is the
presence and duration of organ failure, if organ failure persists for >48 h.
Ref: Gut 2013;62:102–111. doi:10.1136/gutjnl­2012­302779.
Sample Previous Year Question on Pancreatitis based on previous Year Questions of
NEET PG, USMLE,PLAB,FMGE (MCI Screening). Please visit www.medicoapps.org
for more such Quizzes
Areas of peripancreatic fluid seen within the first 4 weeks after onset of interstitial
oedematous pancreatitis is called:
A: Acute Peripancreatic fluid collection
B: Pseudocyst
C: WOPN
D: Abscess
Correct Ans:A
Explanation
APFC (acute peripancreatic fluid collection)
This term applies only to areas of peripancreatic fluid seen within the first 4 weeks
after onset of interstitial oedematous pancreatitis
CECT criteria of peripancreatic fluid collection
In the setting of interstitial oedematous pancreatitis
27/04/2015 neetpgquiz.medicoapps.org/testCreation/preview.php?tid=18
http://neetpgquiz.medicoapps.org/testCreation/preview.php?tid=18 28/61
Homogeneous collection 
Confined by normal peripancreatic  planes
No definable wall encapsulating the collection
Ref: Gut 2013; 62:102–111. doi:10.1136/gutjnl­2012­302779.
Sample Previous Year Question on Pancreatitis based on previous Year Questions of
NEET PG, USMLE,PLAB,FMGE (MCI Screening). Please visit www.medicoapps.org
for more such Quizzes
Ranson’s score is associated with:
A: Volvulus
B: Pancreatitis
C: Appendicitis
D: None of the above
Correct Ans:B
Explanation
Pancreatitis can be classified as severe based on predictors such as APACHE­II scores and
Ranson's signs, and any evidence that the condition is severe mandates care of the
patient in the intensive care unit. Such evidence may take various forms, such as the
onset of encephalopathy, a hematocrit >50%, urine output <50 mL/h, hypotension, fever,
or peritonitis.
Elderly patients with three or more Ranson's criteria should also be monitored carefully
despite the absence of severe pain.
 
Ref: Schwartz’s principle of surgery 9th edition, chapter 33.
 
Sample Previous Year Question on Pancreatitis based on previous Year Questions of
NEET PG, USMLE,PLAB,FMGE (MCI Screening). Please visit www.medicoapps.org
for more such Quizzes
27/04/2015 neetpgquiz.medicoapps.org/testCreation/preview.php?tid=18
http://neetpgquiz.medicoapps.org/testCreation/preview.php?tid=18 29/61
Surgical intervention in acute pancreatitis is indicated in all, EXCEPT:
A: Infected necrosis
B: Infected collection
C: Impacted gallstone in ampulla
D: None of the above
Correct Ans:D
Explanation
Surgical intervention in acute pancreatitis is reserved for patients with infected collections
or infected necrosis only, or to relieve an impacted gallstone in the ampulla if endoscopic
or radiologic treatments are unavailable or unsuccessful. 
 
Ref: Schwartz’s principle of surgery 9th edition, chapter 33.
Sample Previous Year Question on Pancreatitis based on previous Year Questions of
NEET PG, USMLE,PLAB,FMGE (MCI Screening). Please visit www.medicoapps.org
for more such Quizzes
A 58 year old male alcoholic with chronic pancreatitis develops a palpable abdominal
mass. Ultrasound reveals a 9 cm cystic lesion adjacent to the pancreas. An important
complication that might occur if this cyst ruptured would be?
A: Anaphylactic shock
B: Carcinomatosis
C: Disseminated infection
D: Intestinal hemorrhage
27/04/2015 neetpgquiz.medicoapps.org/testCreation/preview.php?tid=18
http://neetpgquiz.medicoapps.org/testCreation/preview.php?tid=18 30/61
Correct Ans:D
Explanation
The patient most likely has a pancreatic pseudocyst, which is a complication of
pancreatitis. Pancreatic pseudocyst is not a true cyst; it is lined by granulation tissue and
collagen. It contains pancreatic juices and lysed blood, so rupture would spill the active
digestive enzymes onto the adjacent viscera, particularly the stomach, small intestine,
and transverse colon. Digestive action produces potentially severe gastrointestinal
hemorrhage.
 
Anaphylactic shock results from massive activation of the IgE­mediated branch of the
immune system. Pancreatic secretions do not elicit an IgE response. The classic
abdominal cyst that ruptures, producing anaphylactic shock, is a hydatid cyst.
 
Carcinomatosis is widespread serosal spread of a carcinoma, typically due to tumor
spillage into a body cavity. Although this may occur with pancreatic mucinous
cystadenocarcinoma, this disease is far less likely to occur than is pancreatic pseudocyst
in a patient with chronic pancreatitis.
 
Pancreatic pseudocyst is not an infective disease. Although septic abscesses do occur in
the abdomen, and may even complicate a pancreatic pseudocyst, the danger of rupture is
more associated with tissue destruction by pancreatic enzymes than with infection.
 
Ref: Fisher W.E., Anderson D.K., Bell R.H., Saluja A.K., Brunicardi F.C. (2010). Chapter 33.
Pancreas. In F.C. Brunicardi, D.K. Andersen, T.R. Billiar, D.L. Dunn, J.G. Hunter, J.B.
Matthews, R.E. Pollock (Eds), Schwartz's Principles of Surgery, 9e.
 
Sample Previous Year Question on Pancreatitis based on previous Year Questions of
NEET PG, USMLE,PLAB,FMGE (MCI Screening). Please visit www.medicoapps.org
for more such Quizzes
A 23 year old female presented with symptoms of severe pancreatitis. All of the following
are the bad prognostic signs in this patient, EXCEPT:
A: TLC> 16000
B: Calcium less than 8 mmol/L
C: Glucose > 200 mg%
27/04/2015 neetpgquiz.medicoapps.org/testCreation/preview.php?tid=18
http://neetpgquiz.medicoapps.org/testCreation/preview.php?tid=18 31/61
D: Prothrombin > 2 times the control
Correct Ans:D
Explanation
Ranson criteria differentiates mild and severe pancreatitis. It includes five parameters
determined at the time of admission and six parameters determined during the
subsequent 48 hours.
 
Ranson Criteria:
 
On admission Within Next 48 h
Age >55 y (>70 y) Decrease in hematocrit by >10% (same)
White blood cell count 
>16,000/mm3 
(>18,000/mm3)
Estimated fluid sequestration >6 L (>4 L)
Blood glucose level >200 
milligrams/dL (>220 
milligrams/dL)
Serum calcium level <8.0 milligrams/dL (same)
Serum lactate dehydrogenase 
level >350 IU/L (>400 IU/L)
Partial pressure of arterial oxygen <60 mm Hg 
(omitted)
Serum aspartate 
aminotransferase level >250 
IU/L (same)
Increase in blood urea nitrogen level >5 
milligrams/dL after IV fluid hydration (>2 
milligrams/dL)
  Base deficit of >4 mmol/L (>6 mmol)
Patients with three or more Ranson criteria have more severe disease and an increased
risk of complications and death.
Ref: Atilla R., Oktay C. (2011). Chapter 82. Pancreatitis and Cholecystitis. In J.E. Tintinalli,
J.S. Stapczynski, D.M. Cline, O.J. Ma, R.K. Cydulka, G.D. Meckler (Eds), Tintinalli's
Emergency Medicine: A Comprehensive Study Guide, 7e.
Sample Previous Year Question on Pancreatitis based on previous Year Questions of
NEET PG, USMLE,PLAB,FMGE (MCI Screening). Please visit www.medicoapps.org
for more such Quizzes
Which of the following is NOT a prognostic factor for acute pancreatitis?
27/04/2015 neetpgquiz.medicoapps.org/testCreation/preview.php?tid=18
http://neetpgquiz.medicoapps.org/testCreation/preview.php?tid=18 32/61
A: Hypocalcemia
B: Hyperglycemia
C: AST elevation
D: Increased serum amylase
Correct Ans:D
Explanation
The prognostic factors considered for predicting the severity of acute pancreatitis are :
Arterial PaO2 less than 60 mmHg
LDH more than 600 IU/L
AST more than 200 IU/L
WBC more than 15,000/mm3
Serum albumin less than 3.2gm/dl
Serum calcium less than 8 mg/dl
Blood glucose more than 180 mg/dl
Blood urea more than 45 mg/dl
Ref: Grading and Staging in Gastroenterology  By Guido N. J.
Tytgat, page 300.
Sample Previous Year Question on Pancreatitis based on previous Year Questions of
NEET PG, USMLE,PLAB,FMGE (MCI Screening). Please visit www.medicoapps.org
for more such Quizzes
Child's Child­Turcotte­Pugh  is used in :
A: Pancreatitis
B: Cirrhosis
C: Multiple myeloma
D: AIDS
27/04/2015 neetpgquiz.medicoapps.org/testCreation/preview.php?tid=18
http://neetpgquiz.medicoapps.org/testCreation/preview.php?tid=18 33/61
Correct Ans:B
Explanation
The Child­Turcotte­Pugh (CTP) score was originally developed to evaluate the risk of
portacaval shunt procedures secondary to portal hypertension and subsequently has been
shown to be useful in predicting surgical risks of other intra­abdominal operations
performed on cirrhotic patients. Numerous studies have demonstrated overall surgical
mortality rates of 10% for patients with class A cirrhosis, 30% for those with class B
cirrhosis, and 75 to 80% for those with class C cirrhosis.
 
Ref : Tsung A. (2010). Chapter 31. Liver. In T.R. Billiar, D.L. Dunn (Eds), Schwartz's
Principles of Surgery, 9e.
Sample Previous Year Question on Pancreatitis based on previous Year Questions of
NEET PG, USMLE,PLAB,FMGE (MCI Screening). Please visit www.medicoapps.org
for more such Quizzes
Gallstones may be complicated by which of the following?
1. Pancreatitis
2. Choledocholithisasis
3. Acute cholecystitis
4. Carcinoma stomach
5. Carcinoma pancreas
A: 1,2 & 3
B: 2,3 & 4
C: 1,3 & 5
D: 2,3 & 5
Correct Ans:A
Explanation
Effects and complications of gallbladder stones are:
In Gallbladder In Bile ducts In the intestine
Silent stone
Obstructive jaundice
Acute intestinal obstruction
27/04/2015 neetpgquiz.medicoapps.org/testCreation/preview.php?tid=18
http://neetpgquiz.medicoapps.org/testCreation/preview.php?tid=18 34/61
Acute cholecystitis
Chronic cholecystitis
Mucocele
Empyema
Perforation
Gangrene
Carcinoma
Cholangitis
Acute pancreatitis
(Gallstone ileus)
Ref: Bailey & Love 25/e, Page 1120.
Sample Previous Year Question on Pancreatitis based on previous Year Questions of
NEET PG, USMLE,PLAB,FMGE (MCI Screening). Please visit www.medicoapps.org
for more such Quizzes
The Reynold's pentad of fever, jaundice, right upper quadrant pain, septic shock and
mental status change in typical of:
A: Cholangitis
B: Hepatitis
C: Cholecystitis
D: Pancreatitis
Correct Ans:A
Explanation
Cholangitis is one of the two main complications of choledochal stones, the other being
gallstone pancreatitis. The most common presentation is fever, epigastric or right upper
quadrant pain, and jaundice. These classic symptoms, well known as Charcot's triad, are
present in about two thirds of patients. 
 
The illness may progress rapidly with septicemia and disorientation, known as Reynolds
pentad (e.g., fever, jaundice, right upper quadrant pain, septic shock, and mental status
changes). 
 
Ref: Oddsdottir M., Pham T.H., Hunter J.G. (2010). Chapter 32. Gallbladder and the
27/04/2015 neetpgquiz.medicoapps.org/testCreation/preview.php?tid=18
http://neetpgquiz.medicoapps.org/testCreation/preview.php?tid=18 35/61
Extrahepatic Biliary System. In F.C. Brunicardi, D.K. Andersen, T.R. Billiar, D.L. Dunn, J.G.
Hunter, J.B. Matthews, R.E. Pollock (Eds), Schwartz's Principles of Surgery, 9e.
Sample Previous Year Question on Pancreatitis based on previous Year Questions of
NEET PG, USMLE,PLAB,FMGE (MCI Screening). Please visit www.medicoapps.org
for more such Quizzes
In case of choledochal cyst, side­to­side choledochoduodenostomy is NOT recommended
because of the risk of:
A: Malignancy
B: Pancreatitis
C: Recurrent Cholangitis
D: Gall stones
Correct Ans:C
Explanation
The treatment for choledochal cyst is complete excision with Roux­en­Y
hepaticojejunostomy. The duodenal end of the bile duct should be oversewn without
injury to the anomalous entry of the pancreatic duct, limiting the amount of residual
biliary tissue at risk for malignancy. Side­to­side choledochoduodenostomy is not
recommended because it is followed by a high incidence of stricture of the anastomosis
and recurrent cholangitis. Cholecystectomy is always performed.
 
Ref: Albanese C.T., Sylvester K.G. (2010). Chapter 43. Pediatric Surgery. In G.M. Doherty
(Ed), CURRENT Diagnosis & Treatment: Surgery, 13e.
Sample Previous Year Question on Pancreatitis based on previous Year Questions of
NEET PG, USMLE,PLAB,FMGE (MCI Screening). Please visit www.medicoapps.org
for more such Quizzes
What will be the likely diagnosis in a thirty year old male presenting with epigastric pain
radiating to back that wakes him up at night and is relieved by consuming food. He has a
past history of surgery for a perforated duodenal ulcer, treated with omental patch and
proton pump inhibitors and analgesics:
27/04/2015 neetpgquiz.medicoapps.org/testCreation/preview.php?tid=18
http://neetpgquiz.medicoapps.org/testCreation/preview.php?tid=18 36/61
A: Atrophic Gastritis
B: Duodenal Ulcer
C: Gastric Ulcer
D: Chronic Pancreatitis
Correct Ans:B
Explanation
 
Epigastric pain relieved by food that awakens the patient of night (when stomach is
empty) suggests a diagnosis of duodenal ulcer. Gastric ulcer pain is exacerbated by food
and usually does not awaken the patient at night. Pancreatic pain is not relieved by food &
is typically relieved by bending forwards & worsened on lying down.
 
Ref: Bailey and Love Short Practice of Surgery 24th Edition, Page 1038­9; Current
Diagnosis & Treatment in Surgery (CSDT) 13th Edition, Page85.
Sample Previous Year Question on Pancreatitis based on previous Year Questions of
NEET PG, USMLE,PLAB,FMGE (MCI Screening). Please visit www.medicoapps.org
for more such Quizzes
Ten days after a splenectomy for blunt abdominal trauma, a 23­year­old man complains of
upper abdominal and lower chest pain exacerbated by deep breathing. He is anorectic but
ambulatory and otherwise making satisfactory progress. On physical examination, his
temperature is 38.2°C (108°C) rectally, and he has decreased breath sounds at the left
lung base. His abdominal wound appears to be healing well, bowel sound are active and
there are no peritoneal signs. Rectal examination is negative. The W.B.C. count is 12,500
mm3 with a shift to left. Chest X­rays show platelike atelectasis of the left lung field.
Abdominal X­rays show a nonspecific gas pattern in the bowel and an air­fluid level in the
left upper quadrant. Serum amylase is 150 Somogyi units dl (normal 60 to 80). The most
likely diagnosis is:
A: Subphrenic abscess
B:
C: Pancreatitis
27/04/2015 neetpgquiz.medicoapps.org/testCreation/preview.php?tid=18
http://neetpgquiz.medicoapps.org/testCreation/preview.php?tid=18 37/61
D: Pulmonary embolism
Correct Ans:A
Explanation
Patient is showing features of subphrenic abcess. Patient presents with local or subcostal pain and 
tenderness, unexplained fever and tachycardia. 
Investigation: CBC shows leucocytosis. 
X­ray chest shows elevated hemidiaphragm, blunting of the costophrenic angle, pleural effusion and 
pulmonary infiltrates or atelectasis. 
CT scan of abdomen is used to confirm the diagnosis. Scanning techniques such as gallium 
technetium subtraction or following injection of gallium labeled leucocytes is also useful in the 
localization of subphrenic abscess. 
Treatment includes surgical drainage and appropriate antibiotic therapy.
Ref: Crofton and Douglas's Respiratory Diseases: 2 Volume Set, 5th Edition By Anthony Seaton, Page 
1239; Common Surgical Emergencies  By S. K. Kochar, Page 139; SRB's Manual of Surgery By Bhat, 
3rd Edition, Page 508
Sample Previous Year Question on Pancreatitis based on previous Year Questions of
NEET PG, USMLE,PLAB,FMGE (MCI Screening). Please visit www.medicoapps.org
for more such Quizzes
In which of the following condition is Alvarado's score indicated?
A: Pancreatitis
B: Appendicitis
C: Cholecystitis
D: Cholangitis
Correct Ans:B
Explanation
27/04/2015 neetpgquiz.medicoapps.org/testCreation/preview.php?tid=18
http://neetpgquiz.medicoapps.org/testCreation/preview.php?tid=18 38/61
Alvarado’s score is a 10 point clinical scoring system designed to predict appendicitis
based on clinical parameters.
Alvarado’s clinical scoring for appendicitis:
 
Clinical parameter Score
Migration of pain 1
Anorexia 1
Nausea or Vomiting  1
Right iliac fossa tenderness 2
Rebound tenderness 1
Temperature more than 37.3 degree celsius 1
WBC more than 10,000 2
Left shift more than 75% PMNs 1
Total 10
An Alvarado score of 8 or greater is about 96% sensitive for appendicitis, while a score
less than 5 rules out the diagnosis.
Ref: Case Studies in Emergency Medicine edited by Rebecca Jeanmonod, page 27
Sample Previous Year Question on Pancreatitis based on previous Year Questions of
NEET PG, USMLE,PLAB,FMGE (MCI Screening). Please visit www.medicoapps.org
for more such Quizzes
Child with hyperammonemia, later develops pancreatitis and basal ganglia stroke. He may
have:
A: Homocystinuria
B: Maple syrup urine disorder
C: Methylmalonic acidemia
27/04/2015 neetpgquiz.medicoapps.org/testCreation/preview.php?tid=18
http://neetpgquiz.medicoapps.org/testCreation/preview.php?tid=18 39/61
D: Tyrosinemia
Correct Ans:C
Explanation
Children with severe methylmalonic acidemia present with acute, life­threatening
metabolic acidosis, hyperammonemia, and bone marrow depression in early infancy or
with metabolic acidosis, vomiting, and failure to thrive during the first few months of life.
Most patients with severe disease have mild or moderate mental retardation. Late
complications include pancreatitis, cardiomyopathy, and basal ganglia stroke, and in
methylmalonic aciduria, interstitial nephritis.
 
Ref: Thomas J.A., Van Hove J.L. (2012). Chapter 36. Inborn Errors of Metabolism. In W.W.
Hay, Jr., M.J. Levin, R.R. Deterding, J.J. Ross, J.M. Sondheimer (Eds), CURRENT Diagnosis
& Treatment: Pediatrics, 21e.
Sample Previous Year Question on Pancreatitis based on previous Year Questions of
NEET PG, USMLE,PLAB,FMGE (MCI Screening). Please visit www.medicoapps.org
for more such Quizzes
A patient undergoes a prolonged and complicated pancreatic surgery for chronic
pancreatitis. Most preferred route for supplementary nutrition in this patient would be:
A: Total Parental Nutrition
B: Feeding Gastrostomy
C: Feeding Jejunostomy
D: Oral feeding
Correct Ans:C
Explanation
When a patient has undergone a prolonged and complicated pancreatic surgery, the
recovery of the patient will require a form of nutrition that gives rest to the pancreas and
protect the pancreatic anastomosis. A feeding jejunostomy in such a case can provide
supplementary nutrition and also gives protection to the pancreatic anastomosis and
cause minimal stimulation of pancreatic secretion thereby giving rest to the pancreas.
27/04/2015 neetpgquiz.medicoapps.org/testCreation/preview.php?tid=18
http://neetpgquiz.medicoapps.org/testCreation/preview.php?tid=18 40/61
 
Ref: Clinical Nutrition in Gastrointestinal Disease By Buchman, 2006, Pages 256­57;
Artificial Nutrition Support in Clinical Practice By Jason Payne, 2nd Edition, Page 273;
Essentials of General Surgery By Lawrence, 4th Edition, Page 80
Sample Previous Year Question on Pancreatitis based on previous Year Questions of
NEET PG, USMLE,PLAB,FMGE (MCI Screening). Please visit www.medicoapps.org
for more such Quizzes
Which of the following is the most common cause of pancreatic pseudocyst?
A: Ca pancreas
B: Pancreatitis
C: Trauma
D: ERCP
Correct Ans:B
Explanation
Pancreatic pseudocysts mostly occur as a complication of acute and chronic pancreatitis. In children, 
abdominal trauma is the most common cause.
Sample Previous Year Question on Pancreatitis based on previous Year Questions of
NEET PG, USMLE,PLAB,FMGE (MCI Screening). Please visit www.medicoapps.org
for more such Quizzes
The immunoglobulin level, which is specifically elevated in 
autoimmune pancreatitis is which of the following?
A: IgM
B: IgE
27/04/2015 neetpgquiz.medicoapps.org/testCreation/preview.php?tid=18
http://neetpgquiz.medicoapps.org/testCreation/preview.php?tid=18 41/61
C: IgG4
D: IgG2
Correct Ans:C
Explanation
Serum IgG4 is elevated at least twofold higher than 135 mg/dL in those 
with autoimmune pancreatitis.
Reference:
Harrisons Principles of Internal Medicine, 18th Edition, Page 2644
Sample Previous Year Question on Pancreatitis based on previous Year Questions of
NEET PG, USMLE,PLAB,FMGE (MCI Screening). Please visit www.medicoapps.org
for more such Quizzes
Cobalamin absorption may be abnormal in all, EXCEPT:
A: Chronic pancreatitis
B: Achlorhydria
C: Bacterial overgrowth syndromes
D: Diverticular disease of colon
Correct Ans:D
Explanation
Cobalamin absorption may be abnormal in:
Pernicious anemia: Due to absence of both gastric acid and intrinsic factor secretion.
Chronic pancreatitis as a result of deficiency of pancreatic proteases to split the
cobalamin–R binder complex.
27/04/2015 neetpgquiz.medicoapps.org/testCreation/preview.php?tid=18
http://neetpgquiz.medicoapps.org/testCreation/preview.php?tid=18 42/61
Achlorhydria or absence of another factor secreted with acid that is responsible for
splitting cobalamin away from the proteins in food to which it is bound
Bacterial overgrowth syndromes: leading to bacterial utilization of cobalamin (often
referred to as stagnant bowel syndrome).
Ileal dysfunction: due to impaired function of the mechanism of cobalamin–intrinsic
factor uptake by ileal intestinal epithelial cells.
Ref: Harrison, Edition­18, e37­1, e37­1t.
Sample Previous Year Question on Pancreatitis based on previous Year Questions of
NEET PG, USMLE,PLAB,FMGE (MCI Screening). Please visit www.medicoapps.org
for more such Quizzes
Pancreatitis, pituitary tumor and phaeochromocytoma may be associated with which of
the following type of thyroid cancer?
A: Anaplastic carcinoma of thyroid
B: Medullary carinoma of thyroid
C: Papillary carcinoma of thyroid
D: Follicular carcinoma of thyroid
Correct Ans:B
Explanation
A MEN 1, or Wermer’s syndrome, is inherited as an autosomal dominant trait. This
syndrome is characterized by neoplasia of the parathyroid glands, enteropancreatic
tumors, anterior pituitary adenomas, and other neuroendocrine tumors with variable
penetrance. Among the types of thyroid cancer given in the question stem, medullary
carcinoma of the thyroid is associated with multiple endocrine neoplasia syndrome.
 
Ref: Harrison’s Internal Medicine, 18th Edition, Chapter 351
 
Sample Previous Year Question on Pancreatitis based on previous Year Questions of
NEET PG, USMLE,PLAB,FMGE (MCI Screening). Please visit www.medicoapps.org
for more such Quizzes
Which is the commonest complication of Mumps in adults ?
27/04/2015 neetpgquiz.medicoapps.org/testCreation/preview.php?tid=18
http://neetpgquiz.medicoapps.org/testCreation/preview.php?tid=18 43/61
A: Encephalitis
B: Orchitis
C: Pancreatitis
D: Carditis
Correct Ans:B
Explanation
Meningoencephalitis is common in children.  Unilateral deafness and thyroiditis are also known 
complication.
Sample Previous Year Question on Pancreatitis based on previous Year Questions of
NEET PG, USMLE,PLAB,FMGE (MCI Screening). Please visit www.medicoapps.org
for more such Quizzes
A 25 year old woman presents with bloody diarrhea and is diagnosed as a case of
Ulcerative colitis. Which of the following condition is not associated with ulcerative colitis?
A: Iritis
B: Pancreatitis
C: Sclerosing cholangitis
D: Ankylosing spondylitis
Correct Ans:B
Explanation
Pancreatitis is not an associated extraintestinal manifestation of ulcerative colitis.
Ulcerative Colitis is associated with:
27/04/2015 neetpgquiz.medicoapps.org/testCreation/preview.php?tid=18
http://neetpgquiz.medicoapps.org/testCreation/preview.php?tid=18 44/61
?   Dermatologic:
?  ?  ?  Erythema nodosum
?  ?  ?  Pyoderma gangrenosum
?   Rheumatologic:
?  ?  ?  Peripheral arthritis
?  ?  ?  Ankylosing spondylitis
?  ?  ?  Sacroiliitis
?   Ocular:
?  ?  ?  Conjunctivitis
?  ?  ?  Anterior uveitis/iritis
?  ?  ?  Episcleritis
?   Hepatobiliary:
?  ?  ?  Hepatic steatosis
?  ?  ?  Fatty liver
?  ?  ?  Cholelithiasis
?  ?  ?  Primary sclerosing cholangitis
?   Urologic:
?  ?  ?  Calculi
?  ?  ?  Ileal bladder fistulas
?   Metabolic bone disorders
?   Thromboembolic disorders
Ref: Harrisons Internal Medicine, 18th Edition, Chapter 295, Pages 2487­2489
Sample Previous Year Question on Pancreatitis based on previous Year Questions of
NEET PG, USMLE,PLAB,FMGE (MCI Screening). Please visit www.medicoapps.org
for more such Quizzes
Maldigestion of protein and fat is manifested in chronic 
pancreatitis only if the damage of pancreatic tissue exceeds?
A: 30 %
B: 50 %
C: 90 %
27/04/2015 neetpgquiz.medicoapps.org/testCreation/preview.php?tid=18
http://neetpgquiz.medicoapps.org/testCreation/preview.php?tid=18 45/61
D: 75 %
Correct Ans:C
Explanation
There is a very large reservoir of pancreatic exocrine function. > 90% of 
the pancreas must be damaged before maldigestion of fat and protein is 
manifested.
Reference:
Harrisons Principles of Internal Medicine, 18th Edition, Page 2629
Sample Previous Year Question on Pancreatitis based on previous Year Questions of
NEET PG, USMLE,PLAB,FMGE (MCI Screening). Please visit www.medicoapps.org
for more such Quizzes
Elevated serum amylase usually returns to normal after 7 days of 
acute pancreatitis. If it remains elevated after 7 days, the possible 
causes include all except:
A: Pancreatic ductal disruption
B: Pancreatic ductal obstruction
C: Pseudocyst formation
D: Chronic calcific pancreatitis
Correct Ans:D
Explanation
In acute pancreatitis, the serum amylase and lipase are elevated within 24 
hours of onset and remains  for 3–7 days. Levels  return to normal within 
7 days unless there is pancreatic ductal disruption, ductal obstruction, or 
pseudocyst formation.
27/04/2015 neetpgquiz.medicoapps.org/testCreation/preview.php?tid=18
http://neetpgquiz.medicoapps.org/testCreation/preview.php?tid=18 46/61
Reference:
Harrisons Principles of Internal Medicine, 18th Edition, Page 2631
Sample Previous Year Question on Pancreatitis based on previous Year Questions of
NEET PG, USMLE,PLAB,FMGE (MCI Screening). Please visit www.medicoapps.org
for more such Quizzes
Risk factors for post ERCP pancreatitis include all, except
A: Minor papilla sphincterotomy
B: Sphincter of Oddi dysfunction
C: Age <60 years
D: Age > 60 years
Correct Ans:D
Explanation
Risk factors for post­ERCP pancreatitis are minor papilla sphincterotomy, 
sphincter of Oddi dysfunction, prior history of post­ERCP pancreatitis, age 
<60 years, >2 contrast injections into the pancreatic duct.
Reference:
Harrisons Principles of Internal Medicine, 18th Edition, Page 2635
Sample Previous Year Question on Pancreatitis based on previous Year Questions of
NEET PG, USMLE,PLAB,FMGE (MCI Screening). Please visit www.medicoapps.org
for more such Quizzes
The genetic factors that can increase the susceptibility and modify 
the severity of pancreatic injury in acute pancreatitis are all, 
except:
27/04/2015 neetpgquiz.medicoapps.org/testCreation/preview.php?tid=18
http://neetpgquiz.medicoapps.org/testCreation/preview.php?tid=18 47/61
A: Cationic trypsinogen mutations
B: Pancreatic secretory trypsin inhibitor
C: CFTR
D: ATP7a gene mutation
Correct Ans:D
Explanation
There are some genetic factors that can increase the susceptibility and/or 
modify the severity of pancreatic injury in acute pancreatitis. They are, 
(1) Cationic trypsinogen mutations
(2) Pancreatic secretory trypsin inhibitor (SPINK1)
(3) CFTR, and
(4) Monocyte chemotactic protein (MCP­1) 
 
Menkes syndrome is caused by mutations in the ATP7A gene.
Reference:
Harrisons Principles of Internal Medicine, 18th Edition, Page 2636
 
Sample Previous Year Question on Pancreatitis based on previous Year Questions of
NEET PG, USMLE,PLAB,FMGE (MCI Screening). Please visit www.medicoapps.org
for more such Quizzes
A 30 year old patient is investigated for acute pancreatitis. Which 
scoring system is used for early prediction of mortality?
A: BISAP
27/04/2015 neetpgquiz.medicoapps.org/testCreation/preview.php?tid=18
http://neetpgquiz.medicoapps.org/testCreation/preview.php?tid=18 48/61
B: Balthazar scoring system
C: Ranson’s
D: APACHE II
Correct Ans:A
Explanation
Early predictors of severity at 48 hours included 3 Ranson's signs
and APACHE II score 8. A recent scoring system for the early
prediction of mortality was developed in acute pancreatitis. This
scoring system known as  the Bedside Index of Severity in Acute
Pancreatitis (BISAP), incorporates five clinical and laboratory
parameters obtained within the first 24 hours of hospitalization.
BUN >25, Impaired mental status, SIRS, Age >60 years, Pleural
effusion on radiography. Presence of three or more of these factors
was associated with increased risk for in­hospital mortality.
Reference:
Harrisons Principles of Internal Medicine, 18th Edition, Page 2639
Sample Previous Year Question on Pancreatitis based on previous Year Questions of
NEET PG, USMLE,PLAB,FMGE (MCI Screening). Please visit www.medicoapps.org
for more such Quizzes
All are indications of elective ERCP with sphincterotomy in acute 
pancreatitis, except:
A: Incipient biliary obstruction
B: Those who are poor candidates for cholecystectomy in biliary pancreatitis
C: Bile duct stones after cholecystectomy
27/04/2015 neetpgquiz.medicoapps.org/testCreation/preview.php?tid=18
http://neetpgquiz.medicoapps.org/testCreation/preview.php?tid=18 49/61
D: Walled off pancreatic necrosis
Correct Ans:D
Explanation
Elective ERCP with sphincterotomy is considered in patients with persistent 
/ incipient biliary obstruction, those who are  poor candidates for 
cholecystectomy, and for those in whom there is strong suspicion for bile 
duct stones after cholecystectomy. ERCP with stent placement is 
indicated for pancreatic ductal disruptions.
Reference:
Harrisons Principles of Internal Medicine, 18th Edition, Page 2640
Sample Previous Year Question on Pancreatitis based on previous Year Questions of
NEET PG, USMLE,PLAB,FMGE (MCI Screening). Please visit www.medicoapps.org
for more such Quizzes
Which of the following is an indication of urgent ERCP in 
pancreatitis?
A: Acute alcoholic pancreatitis
B: Acute necrotising pancreatitis
C: Acute biliary pancreatitis with cholangitis
D: Chronic calcific pancreatitis with pseudocyst
Correct Ans:C
Explanation
Urgent ERCP  is indicated in patients who have severe acute biliary 
pancreatitis with cholangitis.
27/04/2015 neetpgquiz.medicoapps.org/testCreation/preview.php?tid=18
http://neetpgquiz.medicoapps.org/testCreation/preview.php?tid=18 50/61
Reference:
Harrisons Principles of Internal Medicine, 18th Edition, Page 2640
Sample Previous Year Question on Pancreatitis based on previous Year Questions of
NEET PG, USMLE,PLAB,FMGE (MCI Screening). Please visit www.medicoapps.org
for more such Quizzes
Which of the following is the usual presenting symptom of 
pseudocyst in pancreatitis?
A: Abdominal swelling
B: Abdominal pain
C: Persistent vomiting
D: Early satiety and postprandial fullness
Correct Ans:B
Explanation
Pseudocysts are caused by pancreatitis in 90% of cases and by trauma in 
10%. Approximately 85% are located in the body or tail of the pancreas. 
Some patients have two or more pseudocysts. Abdominal pain is the usual 
presenting complaint. 
Reference:
Harrisons Principles of Internal Medicine, 18th Edition, Page 2642
Sample Previous Year Question on Pancreatitis based on previous Year Questions of
NEET PG, USMLE,PLAB,FMGE (MCI Screening). Please visit www.medicoapps.org
for more such Quizzes
Which is the most frequently involved artery in acute pancreatitis 
leading to pseudoaneurysm?
27/04/2015 neetpgquiz.medicoapps.org/testCreation/preview.php?tid=18
http://neetpgquiz.medicoapps.org/testCreation/preview.php?tid=18 51/61
A: Splenic artery
B: Superior pancreatico duodenal artery
C: Inferior pancreaticoduodenal artery
D: Gastroduodenal artery
Correct Ans:A
Explanation
Pseudoaneurysms develop in up to 10% of patients with acute 
pancreatitis. The splenic artery is most frequently involved.
Reference:
Harrisons Principles of Internal Medicine, 18th Edition, Page 2642
Sample Previous Year Question on Pancreatitis based on previous Year Questions of
NEET PG, USMLE,PLAB,FMGE (MCI Screening). Please visit www.medicoapps.org
for more such Quizzes
Purtscher’s retinopathy in acute pancreatitis is due to which of the 
following?
A: Occlusion of the posterior retinal artery with aggregated granulocytes
B: Occlusion of ciliary artery with exudation
C: Central retinal artery occlusion with cotton wool spots and hemorrhage
D: Inflammatory exudates in the retinal vein
Correct Ans:A
27/04/2015 neetpgquiz.medicoapps.org/testCreation/preview.php?tid=18
http://neetpgquiz.medicoapps.org/testCreation/preview.php?tid=18 52/61
Explanation
Purtscher's retinopathy a sudden and severe loss of vision in a patient with 
acute pancreatitis. It is due to occlusion of the posterior retinal artery with 
aggregated granulocytes.
Reference:
Harrisons Principles of Internal Medicine, 18th Edition, Page 2642
Sample Previous Year Question on Pancreatitis based on previous Year Questions of
NEET PG, USMLE,PLAB,FMGE (MCI Screening). Please visit www.medicoapps.org
for more such Quizzes
The immunoglobulin level, which is specifically elevated in 
autoimmune pancreatitis is which of the following?
A: IgM
B: IgE
C: IgG4
D: IgG2
Correct Ans:C
Explanation
Serum IgG4 is elevated at least twofold higher than 135 mg/dL in those 
with autoimmune pancreatitis.
Reference:
Harrisons Principles of Internal Medicine, 18th Edition, Page 2644
Sample Previous Year Question on Pancreatitis based on previous Year Questions of
27/04/2015 neetpgquiz.medicoapps.org/testCreation/preview.php?tid=18
http://neetpgquiz.medicoapps.org/testCreation/preview.php?tid=18 53/61
NEET PG, USMLE,PLAB,FMGE (MCI Screening). Please visit www.medicoapps.org
for more such Quizzes
Medical therapy can be tried in gallstone patients with:
A: Contracted and small non functioning gallbladder
B: Radiolucent gallstones
C: Gallstones of size < 30 mm in diameter
D: Gallstone pancreatitis
Correct Ans:B
Explanation
Ursodeoxycholic acid (UDCA) decreases cholesterol saturation of bile. In carefully
selected patients with functioning gallbladder and with radiolucent stones <10 mm in
diameter, complete dissolution with UDCA can be achieved in 50% of patients within 6
months to 2 years.
Reference: 
Harrisons Principles of Internal Medicine, 18th Edition, Page 2621
 
Sample Previous Year Question on Pancreatitis based on previous Year Questions of
NEET PG, USMLE,PLAB,FMGE (MCI Screening). Please visit www.medicoapps.org
for more such Quizzes
A 14 year old male was presented to the casualty on a saturday night with left sided upper
abdominal pain. Clinical examination revealed massive splenomegaly. He has history of
massive hematemesis 2 year back, and was diagnosed to have Extrahepatic Portal Venous
Obstruction (EHPVO) and bleeding was controlled by ligation of the esophageal varices.
What is the likely diagnosis?
27/04/2015 neetpgquiz.medicoapps.org/testCreation/preview.php?tid=18
http://neetpgquiz.medicoapps.org/testCreation/preview.php?tid=18 54/61
A: Acute pancreatitis
B: Aortic dissection
C: Splenic infarction
D: Intussusception
Correct Ans:C
Explanation
EHPVO is a common cause of portal hypertension in children. It commonly presents with
stable massive variceal bleeding during childhood. Patients have massive splenomegaly.
Splenic infarction and mesenteric vein thrombosis are the two possibilities if EHPVO
develops sudden onset of abdominal pain. Here we have splenic infarction as the answer.
Sample Previous Year Question on Pancreatitis based on previous Year Questions of
NEET PG, USMLE,PLAB,FMGE (MCI Screening). Please visit www.medicoapps.org
for more such Quizzes
A 30 year old male presented with insidious onset of diarrhea, steatorrhea, abdominal
pain, weight loss, migratory large joint arthropathy, fever and dementia. The most
probable diagnosis is:
A: Chronic calcific pancreatitis
B: Whipple's disease
C: Tropical sprue
D: Celiac sprue
Correct Ans:B
Explanation
Whipple’s disease is a chronic multisystem disease caused by tropheryma whipplei
bacteria. Dementia is a late symptom. Diagnosed by PAS positive macrophages in small
27/04/2015 neetpgquiz.medicoapps.org/testCreation/preview.php?tid=18
http://neetpgquiz.medicoapps.org/testCreation/preview.php?tid=18 55/61
intestinal biopsy. DOC is double strength trimethoprim / sulfamethoxazole.  
Ref: Harrisons Principles of Internal Medicine, 18th Edition, Page 2479
Sample Previous Year Question on Pancreatitis based on previous Year Questions of
NEET PG, USMLE,PLAB,FMGE (MCI Screening). Please visit www.medicoapps.org
for more such Quizzes
About autoimmune pancreatitis:
a) It is associated with other autoimmune disorders
b) Obstructive jaundice is a feature
c) Elevated serum IGM 4 is a marker
d) Distinct enlargement of the head of pancreas in CT abdomen
e) Glucocorticoids are effective in alleviating symptoms
A: b,c,d,e ­ True & a­False
B: c,d,e­True & a,b­False
C: a,e­True & b,c,d­False
D: a,b,d, e­True & c­False
Correct Ans:D
Explanation
Elevated IgG4 is a marker of autoimmune pancreatitis. 50­75 patients presents with
obstructive jaundice. Steroids are effective in treatment. CT showed diffuse enlargement,
focal enlargement and distinct enlargement of the head of the pancreas. MRCP reveals
strictures in the bile duct.
Ref: Harrisons , Edition 18 , Page ­ 2644
Sample Previous Year Question on Pancreatitis based on previous Year Questions of
NEET PG, USMLE,PLAB,FMGE (MCI Screening). Please visit www.medicoapps.org
for more such Quizzes
All are important hypersensitivity related side effects of high dose sulfasalazine therapy,
27/04/2015 neetpgquiz.medicoapps.org/testCreation/preview.php?tid=18
http://neetpgquiz.medicoapps.org/testCreation/preview.php?tid=18 56/61
EXCEPT:
A: Hepatitis
B: Agranulocytosis
C: Pancreatitis
D: Thrombocytopenia
Correct Ans:D
Explanation
Although sulfasalazine is more effective at higher doses, up to 30% of patients experience
allergic reactions or intolerable side effects such as headache, anorexia, nausea, and
vomiting that are attributable to the sulfapyridine moiety. 
ALSO KNOW:
Hypersensitivity reactions, independent of sulfapyridine levels, include rash, fever,
hepatitis, agranulocytosis, hypersensitivity pneumonitis, pancreatitis, worsening of colitis,
and reversible sperm abnormalities. 
 
Sulfasalazine can also impair folate absorption, and patients should be given folic acid
supplements.
Ref: Harrison, Edition­18, Page­2489.
Sample Previous Year Question on Pancreatitis based on previous Year Questions of
NEET PG, USMLE,PLAB,FMGE (MCI Screening). Please visit www.medicoapps.org
for more such Quizzes
Cobalamin absorption may be abnormal in all, EXCEPT:
A: Chronic pancreatitis
B: Achlorhydria
27/04/2015 neetpgquiz.medicoapps.org/testCreation/preview.php?tid=18
http://neetpgquiz.medicoapps.org/testCreation/preview.php?tid=18 57/61
C: Bacterial overgrowth syndromes
D: Diverticular disease of colon
Correct Ans:D
Explanation
Cobalamin absorption may be abnormal in:
Pernicious anemia: Due to absence of both gastric acid and intrinsic factor secretion.
Chronic pancreatitis as a result of deficiency of pancreatic proteases to split the
cobalamin–R binder complex.
Achlorhydria or absence of another factor secreted with acid that is responsible for
splitting cobalamin away from the proteins in food to which it is bound
Bacterial overgrowth syndromes: leading to bacterial utilization of cobalamin (often
referred to as stagnant bowel syndrome).
Ileal dysfunction: due to impaired function of the mechanism of cobalamin–intrinsic
factor uptake by ileal intestinal epithelial cells.
Ref: Harrison, Edition­18, e37­1, e37­1t.
Sample Previous Year Question on Pancreatitis based on previous Year Questions of
NEET PG, USMLE,PLAB,FMGE (MCI Screening). Please visit www.medicoapps.org
for more such Quizzes
The modified Mayo score used in clinical trials to assess the severity of:
A: Ulcerative colitis
B: Crohn's disease
C: Diverticulosis
D: Pancreatitis
Correct Ans:A
Explanation
27/04/2015 neetpgquiz.medicoapps.org/testCreation/preview.php?tid=18
http://neetpgquiz.medicoapps.org/testCreation/preview.php?tid=18 58/61
Modifications of the original Truelove and Witts' criteria are used in daily practice, the modified Mayo
score is used in current clinical trials.
The mayo score uses:
Stool frequency 
Rectal bleeding 
Mucosal appearance in endoscopy
Physician's global assessment
 
Ref: Journal of Crohn's and Colitis (2012) 6, 965–990
Sample Previous Year Question on Pancreatitis based on previous Year Questions of
NEET PG, USMLE,PLAB,FMGE (MCI Screening). Please visit www.medicoapps.org
for more such Quizzes
Which of the following is the most rare complication of acute pancreatitis?
A: Hypotension
B: Jaundice
C: Retinopathy
D: Hypertriglyceridemia
Correct Ans:C
Explanation
Due to blockage of the posterior retinal artery by aggregated granulocytes, hemorrhage
and cotton­wool spots are seen in the optic disk and macula in a patient with acute
pancreatitis. Such an unusual complication of acute pancreatitis is known as Purtscher's
retinopathy, which is characterized by sudden and severe loss of vision.
Hypertriglyceridemia is seen in 15% of the patients with acute pancreatitis.
 
Ref: Friedman L.S. (2013). Chapter 16. Liver, Biliary Tract, & Pancreas Disorders. In M.A.
Papadakis, S.J. McPhee, M.W. Rabow (Eds), CURRENT Medical Diagnosis & Treatment
2013.
27/04/2015 neetpgquiz.medicoapps.org/testCreation/preview.php?tid=18
http://neetpgquiz.medicoapps.org/testCreation/preview.php?tid=18 59/61
 
Sample Previous Year Question on Pancreatitis based on previous Year Questions of
NEET PG, USMLE,PLAB,FMGE (MCI Screening). Please visit www.medicoapps.org
for more such Quizzes
What does HbA1c level in the blood indicate?
A: Acute rise of sugar
B: Long terms status of blood sugar
C: Hepatorenal syndrome
D: Chronic pancreatitis
Correct Ans:B
Explanation
Glycosylated Hemoglobin (HbA1C) is the standard method for assessing long­term
glycemic control. When plasma glucose is excessively elevated, it enters the erythrocytes,
and glycates the amino group of lysine residues and the amino terminals of hemoglobin.
This results in the formation of glycosylated hemoglobin. Since the half­life of an
erythrocyte is typically 120 days, the level of glycated hemoglobin (HbA1c) reflects the
mean blood glucose concentration over the preceding 2­3 months. Measurement of HbA1c
therefore provides valuable information for management of diabetes mellitus.
 
Ref: Harrison’s Internal Medicine, 18th Edition, Page 2992 ; Harper's Illustrated
Biochemistry, 28th Edition, Chapter 6 ; Tintinalli's Emergency Medicine : A Comprehensive
Study Guide, 7th Edition, Chapter 218.
Sample Previous Year Question on Pancreatitis based on previous Year Questions of
NEET PG, USMLE,PLAB,FMGE (MCI Screening). Please visit www.medicoapps.org
for more such Quizzes
Pancreatitis, pituitary tumor and phaeochromocytoma may be associated with which of
the following type of thyroid cancer?
27/04/2015 neetpgquiz.medicoapps.org/testCreation/preview.php?tid=18
http://neetpgquiz.medicoapps.org/testCreation/preview.php?tid=18 60/61
A: Anaplastic carcinoma of thyroid
B: Medullary carinoma of thyroid
C: Papillary carcinoma of thyroid
D: Follicular carcinoma of thyroid
Correct Ans:B
Explanation
A MEN 1, or Wermer’s syndrome, is inherited as an autosomal dominant trait. This
syndrome is characterized by neoplasia of the parathyroid glands, enteropancreatic
tumors, anterior pituitary adenomas, and other neuroendocrine tumors with variable
penetrance. Among the types of thyroid cancer given in the question stem, medullary
carcinoma of the thyroid is associated with multiple endocrine neoplasia syndrome.
 
Ref: Harrison’s Internal Medicine, 18th Edition, Chapter 351
 
Sample Previous Year Question on Pancreatitis based on previous Year Questions of
NEET PG, USMLE,PLAB,FMGE (MCI Screening). Please visit www.medicoapps.org
for more such Quizzes
The  clinical situations in which intestinal absorption is increased among the following?
A: Wilson disease
B: Pernicious disease
C: Chronic calcific pancreatitis
D: Cystic fibrosis
Correct Ans:A
Explanation
Almost all clinical problems are associated with diminished intestinal absorption of one or
27/04/2015 neetpgquiz.medicoapps.org/testCreation/preview.php?tid=18
http://neetpgquiz.medicoapps.org/testCreation/preview.php?tid=18 61/61
more dietary nutrients. The only clinical situations in which absorption is increased
is Wilson disease, in which absorption of copper is increased.
ALSO NOTE:
Another example is hemochromatosis in which iron absorption is increased.
Ref: Harrison, E­18, P­2460.
Sample Previous Year Question on Pancreatitis based on previous Year Questions of
NEET PG, USMLE,PLAB,FMGE (MCI Screening). Please visit www.medicoapps.org
for more such Quizzes

More Related Content

What's hot

Crohn's disease sample mcq
Crohn's disease sample mcq Crohn's disease sample mcq
Crohn's disease sample mcq Medico Apps
 
Extra hepatic portal vein obstruction
Extra hepatic portal vein obstructionExtra hepatic portal vein obstruction
Extra hepatic portal vein obstructionPratap Tiwari
 
Previous year question on hepatomegaly based on neet pg, usmle, plab and fmge...
Previous year question on hepatomegaly based on neet pg, usmle, plab and fmge...Previous year question on hepatomegaly based on neet pg, usmle, plab and fmge...
Previous year question on hepatomegaly based on neet pg, usmle, plab and fmge...Medico Apps
 
Gastrointestinal mcq
Gastrointestinal mcqGastrointestinal mcq
Gastrointestinal mcqRashed Hassen
 
Groove pancreatitis final.pptx
Groove pancreatitis final.pptxGroove pancreatitis final.pptx
Groove pancreatitis final.pptxUmashankar U S
 
9 renal transplant
9 renal transplant9 renal transplant
9 renal transplantHabrol Afzam
 
Gastroenterology SCE MCQ
Gastroenterology SCE MCQGastroenterology SCE MCQ
Gastroenterology SCE MCQjuuraju
 
500 single best answers in medicine
500 single best answers in medicine500 single best answers in medicine
500 single best answers in medicinehamadadodo
 
cases and treatment of peptic ulcers
cases and treatment of peptic ulcerscases and treatment of peptic ulcers
cases and treatment of peptic ulcersMjnoOntk Ana
 
Jaundice and LFT interpretation
Jaundice and LFT interpretationJaundice and LFT interpretation
Jaundice and LFT interpretationAnahita Sharma
 
Primary sclerosing cholangitis
Primary sclerosing cholangitisPrimary sclerosing cholangitis
Primary sclerosing cholangitisJustin V Sebastian
 
Biological therapy for Ulcerative colitis
Biological therapy for Ulcerative colitisBiological therapy for Ulcerative colitis
Biological therapy for Ulcerative colitisDr Amit Dangi
 

What's hot (20)

Crohn's disease sample mcq
Crohn's disease sample mcq Crohn's disease sample mcq
Crohn's disease sample mcq
 
Extra hepatic portal vein obstruction
Extra hepatic portal vein obstructionExtra hepatic portal vein obstruction
Extra hepatic portal vein obstruction
 
NAFLD
NAFLDNAFLD
NAFLD
 
Acute cholangitis
Acute cholangitisAcute cholangitis
Acute cholangitis
 
Previous year question on hepatomegaly based on neet pg, usmle, plab and fmge...
Previous year question on hepatomegaly based on neet pg, usmle, plab and fmge...Previous year question on hepatomegaly based on neet pg, usmle, plab and fmge...
Previous year question on hepatomegaly based on neet pg, usmle, plab and fmge...
 
Biliary Atresia
Biliary AtresiaBiliary Atresia
Biliary Atresia
 
Gastrointestinal mcq
Gastrointestinal mcqGastrointestinal mcq
Gastrointestinal mcq
 
Acute appendicitis / RLQ Pain
Acute appendicitis / RLQ PainAcute appendicitis / RLQ Pain
Acute appendicitis / RLQ Pain
 
Pathology of Upper GIT - Quiz
Pathology of Upper GIT - QuizPathology of Upper GIT - Quiz
Pathology of Upper GIT - Quiz
 
Groove pancreatitis final.pptx
Groove pancreatitis final.pptxGroove pancreatitis final.pptx
Groove pancreatitis final.pptx
 
9 renal transplant
9 renal transplant9 renal transplant
9 renal transplant
 
Gastroenterology SCE MCQ
Gastroenterology SCE MCQGastroenterology SCE MCQ
Gastroenterology SCE MCQ
 
baveno 7.pptx
baveno 7.pptxbaveno 7.pptx
baveno 7.pptx
 
500 single best answers in medicine
500 single best answers in medicine500 single best answers in medicine
500 single best answers in medicine
 
Carcinoid tumors
Carcinoid tumorsCarcinoid tumors
Carcinoid tumors
 
Eosinophilic Esophagitis
 Eosinophilic Esophagitis Eosinophilic Esophagitis
Eosinophilic Esophagitis
 
cases and treatment of peptic ulcers
cases and treatment of peptic ulcerscases and treatment of peptic ulcers
cases and treatment of peptic ulcers
 
Jaundice and LFT interpretation
Jaundice and LFT interpretationJaundice and LFT interpretation
Jaundice and LFT interpretation
 
Primary sclerosing cholangitis
Primary sclerosing cholangitisPrimary sclerosing cholangitis
Primary sclerosing cholangitis
 
Biological therapy for Ulcerative colitis
Biological therapy for Ulcerative colitisBiological therapy for Ulcerative colitis
Biological therapy for Ulcerative colitis
 

Viewers also liked

Krok 1 2014 - biochemistry
Krok 1   2014 - biochemistryKrok 1   2014 - biochemistry
Krok 1 2014 - biochemistryEneutron
 
իմ քաղաքը` թումոյի այգի
իմ քաղաքը` թումոյի այգիիմ քաղաքը` թումոյի այգի
իմ քաղաքը` թումոյի այգիAshkhen Grigoryan
 
Wastek Teknologi (1)
Wastek Teknologi (1)Wastek Teknologi (1)
Wastek Teknologi (1)Tedi Eka
 
The fundamentals that directly mean to affect conversion rate of mobile app
The fundamentals that directly mean to affect conversion rate of mobile appThe fundamentals that directly mean to affect conversion rate of mobile app
The fundamentals that directly mean to affect conversion rate of mobile appiMOBDEV Technologies Pvt. Ltd.
 
14 - Revisión de las proyecciones de población, parte 3 (2015) (ESP)
14 - Revisión de las proyecciones de población, parte 3 (2015) (ESP)14 - Revisión de las proyecciones de población, parte 3 (2015) (ESP)
14 - Revisión de las proyecciones de población, parte 3 (2015) (ESP)InstitutoBBVAdePensiones
 
Sisältömarkkinointi on verkkokauppiaan salainen ase
Sisältömarkkinointi on verkkokauppiaan salainen aseSisältömarkkinointi on verkkokauppiaan salainen ase
Sisältömarkkinointi on verkkokauppiaan salainen aseKati Keronen
 
Amr Training Certificates - 2002-2005-2010
Amr Training Certificates - 2002-2005-2010Amr Training Certificates - 2002-2005-2010
Amr Training Certificates - 2002-2005-2010Amr Sakran
 
Model discovery learning
Model discovery learningModel discovery learning
Model discovery learningMuhammad Fikri
 
Fault presentation by muneeb
Fault presentation by muneebFault presentation by muneeb
Fault presentation by muneebMuneeb ur Rehman
 
Employee of the month
Employee of the monthEmployee of the month
Employee of the monthAdam Chard
 
April 2016 - Market Snapshot - Santa Clara County
April 2016 - Market Snapshot - Santa Clara CountyApril 2016 - Market Snapshot - Santa Clara County
April 2016 - Market Snapshot - Santa Clara CountyMLSListings Inc
 
Práctica3 propiedades mecanicas_alvarogarciacamaron
Práctica3 propiedades mecanicas_alvarogarciacamaronPráctica3 propiedades mecanicas_alvarogarciacamaron
Práctica3 propiedades mecanicas_alvarogarciacamaronAlvarogarcy
 

Viewers also liked (18)

Kamal FMGE MCQ Book
Kamal FMGE MCQ BookKamal FMGE MCQ Book
Kamal FMGE MCQ Book
 
Krok 1 2014 - biochemistry
Krok 1   2014 - biochemistryKrok 1   2014 - biochemistry
Krok 1 2014 - biochemistry
 
My job2
My job2My job2
My job2
 
Silabus bhs inggris wajib kls 11
Silabus bhs inggris wajib kls 11Silabus bhs inggris wajib kls 11
Silabus bhs inggris wajib kls 11
 
իմ քաղաքը` թումոյի այգի
իմ քաղաքը` թումոյի այգիիմ քաղաքը` թումոյի այգի
իմ քաղաքը` թումոյի այգի
 
Wastek Teknologi (1)
Wastek Teknologi (1)Wastek Teknologi (1)
Wastek Teknologi (1)
 
The fundamentals that directly mean to affect conversion rate of mobile app
The fundamentals that directly mean to affect conversion rate of mobile appThe fundamentals that directly mean to affect conversion rate of mobile app
The fundamentals that directly mean to affect conversion rate of mobile app
 
LATIHAN BAB 6
LATIHAN BAB 6LATIHAN BAB 6
LATIHAN BAB 6
 
14 - Revisión de las proyecciones de población, parte 3 (2015) (ESP)
14 - Revisión de las proyecciones de población, parte 3 (2015) (ESP)14 - Revisión de las proyecciones de población, parte 3 (2015) (ESP)
14 - Revisión de las proyecciones de población, parte 3 (2015) (ESP)
 
Sisältömarkkinointi on verkkokauppiaan salainen ase
Sisältömarkkinointi on verkkokauppiaan salainen aseSisältömarkkinointi on verkkokauppiaan salainen ase
Sisältömarkkinointi on verkkokauppiaan salainen ase
 
Arte en vivo
Arte en vivoArte en vivo
Arte en vivo
 
Amr Training Certificates - 2002-2005-2010
Amr Training Certificates - 2002-2005-2010Amr Training Certificates - 2002-2005-2010
Amr Training Certificates - 2002-2005-2010
 
Model discovery learning
Model discovery learningModel discovery learning
Model discovery learning
 
Fault presentation by muneeb
Fault presentation by muneebFault presentation by muneeb
Fault presentation by muneeb
 
Employee of the month
Employee of the monthEmployee of the month
Employee of the month
 
April 2016 - Market Snapshot - Santa Clara County
April 2016 - Market Snapshot - Santa Clara CountyApril 2016 - Market Snapshot - Santa Clara County
April 2016 - Market Snapshot - Santa Clara County
 
Práctica3 propiedades mecanicas_alvarogarciacamaron
Práctica3 propiedades mecanicas_alvarogarciacamaronPráctica3 propiedades mecanicas_alvarogarciacamaron
Práctica3 propiedades mecanicas_alvarogarciacamaron
 
La+NiñEz+..[1]
La+NiñEz+..[1]La+NiñEz+..[1]
La+NiñEz+..[1]
 

Similar to Pancreatitis sample questions based on neet pg , usmle, plab and fmge pattern (mci screening)

Radiology most important signs sample questions based on neet pg , usmle, pla...
Radiology most important signs sample questions based on neet pg , usmle, pla...Radiology most important signs sample questions based on neet pg , usmle, pla...
Radiology most important signs sample questions based on neet pg , usmle, pla...Medico Apps
 
23205039
2320503923205039
23205039radgirl
 
23204995
2320499523204995
23204995radgirl
 
acute pancreatitis a thorough review
 acute pancreatitis a thorough review acute pancreatitis a thorough review
acute pancreatitis a thorough reviewShailesh Gupta
 
23205045
2320504523205045
23205045radgirl
 
Second Year Surgery Case Presentation
Second Year Surgery Case PresentationSecond Year Surgery Case Presentation
Second Year Surgery Case Presentationjnpeacoc
 
23205016
2320501623205016
23205016radgirl
 
Annular Pancreas: An Unusual Presentation
Annular Pancreas: An Unusual PresentationAnnular Pancreas: An Unusual Presentation
Annular Pancreas: An Unusual PresentationApollo Hospitals
 
Gastroenterology ppt 2
Gastroenterology ppt  2Gastroenterology ppt  2
Gastroenterology ppt 2UPUL UDAYARAJ
 
Acs0509 Tumors Of The Pancreas, Biliary Tract, And Liver 2009
Acs0509 Tumors Of The Pancreas, Biliary Tract, And Liver 2009Acs0509 Tumors Of The Pancreas, Biliary Tract, And Liver 2009
Acs0509 Tumors Of The Pancreas, Biliary Tract, And Liver 2009medbookonline
 
Evaluation and management of intestinal obstruction
Evaluation and management of intestinal obstructionEvaluation and management of intestinal obstruction
Evaluation and management of intestinal obstructionImad Zoukar
 
SEVERE ACUTE PANCREATITIS PRESENTATION 2020
SEVERE ACUTE PANCREATITIS PRESENTATION 2020SEVERE ACUTE PANCREATITIS PRESENTATION 2020
SEVERE ACUTE PANCREATITIS PRESENTATION 2020karanchhabra75
 
Caris Centers of Excellence Virtual Molecular Tumor Board - August 12, 2015
Caris Centers of Excellence Virtual Molecular Tumor Board - August 12, 2015Caris Centers of Excellence Virtual Molecular Tumor Board - August 12, 2015
Caris Centers of Excellence Virtual Molecular Tumor Board - August 12, 2015Caris Life Sciences
 
23204973
2320497323204973
23204973radgirl
 
Oscar Imventarza - Argentina - Tuesday 29 - Organ Allocation Optimizing dono...
Oscar Imventarza  - Argentina - Tuesday 29 - Organ Allocation Optimizing dono...Oscar Imventarza  - Argentina - Tuesday 29 - Organ Allocation Optimizing dono...
Oscar Imventarza - Argentina - Tuesday 29 - Organ Allocation Optimizing dono...incucai_isodp
 
Kastenberg 2013 clinics-in-perinatology
Kastenberg 2013 clinics-in-perinatologyKastenberg 2013 clinics-in-perinatology
Kastenberg 2013 clinics-in-perinatologyMOHAMMAD QUAYYUM
 

Similar to Pancreatitis sample questions based on neet pg , usmle, plab and fmge pattern (mci screening) (20)

Radiology most important signs sample questions based on neet pg , usmle, pla...
Radiology most important signs sample questions based on neet pg , usmle, pla...Radiology most important signs sample questions based on neet pg , usmle, pla...
Radiology most important signs sample questions based on neet pg , usmle, pla...
 
23205039
2320503923205039
23205039
 
23204995
2320499523204995
23204995
 
acute pancreatitis a thorough review
 acute pancreatitis a thorough review acute pancreatitis a thorough review
acute pancreatitis a thorough review
 
23205045
2320504523205045
23205045
 
Second Year Surgery Case Presentation
Second Year Surgery Case PresentationSecond Year Surgery Case Presentation
Second Year Surgery Case Presentation
 
23205016
2320501623205016
23205016
 
GI and Liver Malignancies
GI and Liver MalignanciesGI and Liver Malignancies
GI and Liver Malignancies
 
Annular Pancreas: An Unusual Presentation
Annular Pancreas: An Unusual PresentationAnnular Pancreas: An Unusual Presentation
Annular Pancreas: An Unusual Presentation
 
Gastroenterology ppt 2
Gastroenterology ppt  2Gastroenterology ppt  2
Gastroenterology ppt 2
 
Paraneoplastic Pseudoachalasia Secondary to Ovarian Carcinoma: a Hard Pill to...
Paraneoplastic Pseudoachalasia Secondary to Ovarian Carcinoma: a Hard Pill to...Paraneoplastic Pseudoachalasia Secondary to Ovarian Carcinoma: a Hard Pill to...
Paraneoplastic Pseudoachalasia Secondary to Ovarian Carcinoma: a Hard Pill to...
 
Acs0509 Tumors Of The Pancreas, Biliary Tract, And Liver 2009
Acs0509 Tumors Of The Pancreas, Biliary Tract, And Liver 2009Acs0509 Tumors Of The Pancreas, Biliary Tract, And Liver 2009
Acs0509 Tumors Of The Pancreas, Biliary Tract, And Liver 2009
 
Evaluation and management of intestinal obstruction
Evaluation and management of intestinal obstructionEvaluation and management of intestinal obstruction
Evaluation and management of intestinal obstruction
 
SEVERE ACUTE PANCREATITIS PRESENTATION 2020
SEVERE ACUTE PANCREATITIS PRESENTATION 2020SEVERE ACUTE PANCREATITIS PRESENTATION 2020
SEVERE ACUTE PANCREATITIS PRESENTATION 2020
 
Acute pancreatitis nejm 2006
Acute pancreatitis nejm 2006Acute pancreatitis nejm 2006
Acute pancreatitis nejm 2006
 
Caris Centers of Excellence Virtual Molecular Tumor Board - August 12, 2015
Caris Centers of Excellence Virtual Molecular Tumor Board - August 12, 2015Caris Centers of Excellence Virtual Molecular Tumor Board - August 12, 2015
Caris Centers of Excellence Virtual Molecular Tumor Board - August 12, 2015
 
23204973
2320497323204973
23204973
 
Oscar Imventarza - Argentina - Tuesday 29 - Organ Allocation Optimizing dono...
Oscar Imventarza  - Argentina - Tuesday 29 - Organ Allocation Optimizing dono...Oscar Imventarza  - Argentina - Tuesday 29 - Organ Allocation Optimizing dono...
Oscar Imventarza - Argentina - Tuesday 29 - Organ Allocation Optimizing dono...
 
10.1007_s11605-016-3123-1 YST
10.1007_s11605-016-3123-1 YST10.1007_s11605-016-3123-1 YST
10.1007_s11605-016-3123-1 YST
 
Kastenberg 2013 clinics-in-perinatology
Kastenberg 2013 clinics-in-perinatologyKastenberg 2013 clinics-in-perinatology
Kastenberg 2013 clinics-in-perinatology
 

More from Medico Apps

Previous year question on rabies based on neet pg, usmle, plab and fmge or mc...
Previous year question on rabies based on neet pg, usmle, plab and fmge or mc...Previous year question on rabies based on neet pg, usmle, plab and fmge or mc...
Previous year question on rabies based on neet pg, usmle, plab and fmge or mc...Medico Apps
 
Previous year question on medulloblastoma based on neet pg, usmle, plab and f...
Previous year question on medulloblastoma based on neet pg, usmle, plab and f...Previous year question on medulloblastoma based on neet pg, usmle, plab and f...
Previous year question on medulloblastoma based on neet pg, usmle, plab and f...Medico Apps
 
Previous year question on cataract based on neet pg, usmle, plab and fmge or ...
Previous year question on cataract based on neet pg, usmle, plab and fmge or ...Previous year question on cataract based on neet pg, usmle, plab and fmge or ...
Previous year question on cataract based on neet pg, usmle, plab and fmge or ...Medico Apps
 
Previous year question on pharyngeal arches embryology based on neet pg, usml...
Previous year question on pharyngeal arches embryology based on neet pg, usml...Previous year question on pharyngeal arches embryology based on neet pg, usml...
Previous year question on pharyngeal arches embryology based on neet pg, usml...Medico Apps
 
Previous year question on pemphigus vulgaris based on neet pg, usmle, plab an...
Previous year question on pemphigus vulgaris based on neet pg, usmle, plab an...Previous year question on pemphigus vulgaris based on neet pg, usmle, plab an...
Previous year question on pemphigus vulgaris based on neet pg, usmle, plab an...Medico Apps
 
Previous year question on leptospirosis based on neet pg, usmle, plab and fmg...
Previous year question on leptospirosis based on neet pg, usmle, plab and fmg...Previous year question on leptospirosis based on neet pg, usmle, plab and fmg...
Previous year question on leptospirosis based on neet pg, usmle, plab and fmg...Medico Apps
 
Previous year question on glycolysis based on neet pg, usmle, plab and fmge o...
Previous year question on glycolysis based on neet pg, usmle, plab and fmge o...Previous year question on glycolysis based on neet pg, usmle, plab and fmge o...
Previous year question on glycolysis based on neet pg, usmle, plab and fmge o...Medico Apps
 
Previous year question on glycolysis based on neet pg, usmle, plab and fmge o...
Previous year question on glycolysis based on neet pg, usmle, plab and fmge o...Previous year question on glycolysis based on neet pg, usmle, plab and fmge o...
Previous year question on glycolysis based on neet pg, usmle, plab and fmge o...Medico Apps
 
Previous year question on bone cyst based on neet pg, usmle, plab and fmge or...
Previous year question on bone cyst based on neet pg, usmle, plab and fmge or...Previous year question on bone cyst based on neet pg, usmle, plab and fmge or...
Previous year question on bone cyst based on neet pg, usmle, plab and fmge or...Medico Apps
 
Previous year question on ketamine based on neet pg, usmle, plab and fmge or ...
Previous year question on ketamine based on neet pg, usmle, plab and fmge or ...Previous year question on ketamine based on neet pg, usmle, plab and fmge or ...
Previous year question on ketamine based on neet pg, usmle, plab and fmge or ...Medico Apps
 
Ketamine (anaesthesia )sample questions based on neet pg , usmle, plab and fm...
Ketamine (anaesthesia )sample questions based on neet pg , usmle, plab and fm...Ketamine (anaesthesia )sample questions based on neet pg , usmle, plab and fm...
Ketamine (anaesthesia )sample questions based on neet pg , usmle, plab and fm...Medico Apps
 
Hiv aids sample questions based on neet pg , usmle, plab and fmge pattern (mc...
Hiv aids sample questions based on neet pg , usmle, plab and fmge pattern (mc...Hiv aids sample questions based on neet pg , usmle, plab and fmge pattern (mc...
Hiv aids sample questions based on neet pg , usmle, plab and fmge pattern (mc...Medico Apps
 
Hiv aids sample questions based on neet pg , usmle, plab and fmge pattern (mc...
Hiv aids sample questions based on neet pg , usmle, plab and fmge pattern (mc...Hiv aids sample questions based on neet pg , usmle, plab and fmge pattern (mc...
Hiv aids sample questions based on neet pg , usmle, plab and fmge pattern (mc...Medico Apps
 
Hiv aids sample questions based on neet pg , usmle, plab and fmge pattern (mc...
Hiv aids sample questions based on neet pg , usmle, plab and fmge pattern (mc...Hiv aids sample questions based on neet pg , usmle, plab and fmge pattern (mc...
Hiv aids sample questions based on neet pg , usmle, plab and fmge pattern (mc...Medico Apps
 
Hiv aids sample questions based on neet pg , usmle, plab and fmge pattern (mc...
Hiv aids sample questions based on neet pg , usmle, plab and fmge pattern (mc...Hiv aids sample questions based on neet pg , usmle, plab and fmge pattern (mc...
Hiv aids sample questions based on neet pg , usmle, plab and fmge pattern (mc...Medico Apps
 
Hiv aids sample questions based on neet pg , usmle, plab and fmge pattern (mc...
Hiv aids sample questions based on neet pg , usmle, plab and fmge pattern (mc...Hiv aids sample questions based on neet pg , usmle, plab and fmge pattern (mc...
Hiv aids sample questions based on neet pg , usmle, plab and fmge pattern (mc...Medico Apps
 
Meconium stained baby sample questions based on neet pg , usmle, plab and fmg...
Meconium stained baby sample questions based on neet pg , usmle, plab and fmg...Meconium stained baby sample questions based on neet pg , usmle, plab and fmg...
Meconium stained baby sample questions based on neet pg , usmle, plab and fmg...Medico Apps
 
Alzheimers disease sample questions based on neet pg , usmle, plab and fmge p...
Alzheimers disease sample questions based on neet pg , usmle, plab and fmge p...Alzheimers disease sample questions based on neet pg , usmle, plab and fmge p...
Alzheimers disease sample questions based on neet pg , usmle, plab and fmge p...Medico Apps
 
Cystic fibrosis sample questions based on neet pg , usmle, plab and fmge patt...
Cystic fibrosis sample questions based on neet pg , usmle, plab and fmge patt...Cystic fibrosis sample questions based on neet pg , usmle, plab and fmge patt...
Cystic fibrosis sample questions based on neet pg , usmle, plab and fmge patt...Medico Apps
 
Phrenic nerve sample questions based on neet pg , usmle, plab and fmge patter...
Phrenic nerve sample questions based on neet pg , usmle, plab and fmge patter...Phrenic nerve sample questions based on neet pg , usmle, plab and fmge patter...
Phrenic nerve sample questions based on neet pg , usmle, plab and fmge patter...Medico Apps
 

More from Medico Apps (20)

Previous year question on rabies based on neet pg, usmle, plab and fmge or mc...
Previous year question on rabies based on neet pg, usmle, plab and fmge or mc...Previous year question on rabies based on neet pg, usmle, plab and fmge or mc...
Previous year question on rabies based on neet pg, usmle, plab and fmge or mc...
 
Previous year question on medulloblastoma based on neet pg, usmle, plab and f...
Previous year question on medulloblastoma based on neet pg, usmle, plab and f...Previous year question on medulloblastoma based on neet pg, usmle, plab and f...
Previous year question on medulloblastoma based on neet pg, usmle, plab and f...
 
Previous year question on cataract based on neet pg, usmle, plab and fmge or ...
Previous year question on cataract based on neet pg, usmle, plab and fmge or ...Previous year question on cataract based on neet pg, usmle, plab and fmge or ...
Previous year question on cataract based on neet pg, usmle, plab and fmge or ...
 
Previous year question on pharyngeal arches embryology based on neet pg, usml...
Previous year question on pharyngeal arches embryology based on neet pg, usml...Previous year question on pharyngeal arches embryology based on neet pg, usml...
Previous year question on pharyngeal arches embryology based on neet pg, usml...
 
Previous year question on pemphigus vulgaris based on neet pg, usmle, plab an...
Previous year question on pemphigus vulgaris based on neet pg, usmle, plab an...Previous year question on pemphigus vulgaris based on neet pg, usmle, plab an...
Previous year question on pemphigus vulgaris based on neet pg, usmle, plab an...
 
Previous year question on leptospirosis based on neet pg, usmle, plab and fmg...
Previous year question on leptospirosis based on neet pg, usmle, plab and fmg...Previous year question on leptospirosis based on neet pg, usmle, plab and fmg...
Previous year question on leptospirosis based on neet pg, usmle, plab and fmg...
 
Previous year question on glycolysis based on neet pg, usmle, plab and fmge o...
Previous year question on glycolysis based on neet pg, usmle, plab and fmge o...Previous year question on glycolysis based on neet pg, usmle, plab and fmge o...
Previous year question on glycolysis based on neet pg, usmle, plab and fmge o...
 
Previous year question on glycolysis based on neet pg, usmle, plab and fmge o...
Previous year question on glycolysis based on neet pg, usmle, plab and fmge o...Previous year question on glycolysis based on neet pg, usmle, plab and fmge o...
Previous year question on glycolysis based on neet pg, usmle, plab and fmge o...
 
Previous year question on bone cyst based on neet pg, usmle, plab and fmge or...
Previous year question on bone cyst based on neet pg, usmle, plab and fmge or...Previous year question on bone cyst based on neet pg, usmle, plab and fmge or...
Previous year question on bone cyst based on neet pg, usmle, plab and fmge or...
 
Previous year question on ketamine based on neet pg, usmle, plab and fmge or ...
Previous year question on ketamine based on neet pg, usmle, plab and fmge or ...Previous year question on ketamine based on neet pg, usmle, plab and fmge or ...
Previous year question on ketamine based on neet pg, usmle, plab and fmge or ...
 
Ketamine (anaesthesia )sample questions based on neet pg , usmle, plab and fm...
Ketamine (anaesthesia )sample questions based on neet pg , usmle, plab and fm...Ketamine (anaesthesia )sample questions based on neet pg , usmle, plab and fm...
Ketamine (anaesthesia )sample questions based on neet pg , usmle, plab and fm...
 
Hiv aids sample questions based on neet pg , usmle, plab and fmge pattern (mc...
Hiv aids sample questions based on neet pg , usmle, plab and fmge pattern (mc...Hiv aids sample questions based on neet pg , usmle, plab and fmge pattern (mc...
Hiv aids sample questions based on neet pg , usmle, plab and fmge pattern (mc...
 
Hiv aids sample questions based on neet pg , usmle, plab and fmge pattern (mc...
Hiv aids sample questions based on neet pg , usmle, plab and fmge pattern (mc...Hiv aids sample questions based on neet pg , usmle, plab and fmge pattern (mc...
Hiv aids sample questions based on neet pg , usmle, plab and fmge pattern (mc...
 
Hiv aids sample questions based on neet pg , usmle, plab and fmge pattern (mc...
Hiv aids sample questions based on neet pg , usmle, plab and fmge pattern (mc...Hiv aids sample questions based on neet pg , usmle, plab and fmge pattern (mc...
Hiv aids sample questions based on neet pg , usmle, plab and fmge pattern (mc...
 
Hiv aids sample questions based on neet pg , usmle, plab and fmge pattern (mc...
Hiv aids sample questions based on neet pg , usmle, plab and fmge pattern (mc...Hiv aids sample questions based on neet pg , usmle, plab and fmge pattern (mc...
Hiv aids sample questions based on neet pg , usmle, plab and fmge pattern (mc...
 
Hiv aids sample questions based on neet pg , usmle, plab and fmge pattern (mc...
Hiv aids sample questions based on neet pg , usmle, plab and fmge pattern (mc...Hiv aids sample questions based on neet pg , usmle, plab and fmge pattern (mc...
Hiv aids sample questions based on neet pg , usmle, plab and fmge pattern (mc...
 
Meconium stained baby sample questions based on neet pg , usmle, plab and fmg...
Meconium stained baby sample questions based on neet pg , usmle, plab and fmg...Meconium stained baby sample questions based on neet pg , usmle, plab and fmg...
Meconium stained baby sample questions based on neet pg , usmle, plab and fmg...
 
Alzheimers disease sample questions based on neet pg , usmle, plab and fmge p...
Alzheimers disease sample questions based on neet pg , usmle, plab and fmge p...Alzheimers disease sample questions based on neet pg , usmle, plab and fmge p...
Alzheimers disease sample questions based on neet pg , usmle, plab and fmge p...
 
Cystic fibrosis sample questions based on neet pg , usmle, plab and fmge patt...
Cystic fibrosis sample questions based on neet pg , usmle, plab and fmge patt...Cystic fibrosis sample questions based on neet pg , usmle, plab and fmge patt...
Cystic fibrosis sample questions based on neet pg , usmle, plab and fmge patt...
 
Phrenic nerve sample questions based on neet pg , usmle, plab and fmge patter...
Phrenic nerve sample questions based on neet pg , usmle, plab and fmge patter...Phrenic nerve sample questions based on neet pg , usmle, plab and fmge patter...
Phrenic nerve sample questions based on neet pg , usmle, plab and fmge patter...
 

Recently uploaded

VIP Call Girls Pune Vrinda 9907093804 Short 1500 Night 6000 Best call girls S...
VIP Call Girls Pune Vrinda 9907093804 Short 1500 Night 6000 Best call girls S...VIP Call Girls Pune Vrinda 9907093804 Short 1500 Night 6000 Best call girls S...
VIP Call Girls Pune Vrinda 9907093804 Short 1500 Night 6000 Best call girls S...Miss joya
 
VIP Call Girls Pune Sanjana 9907093804 Short 1500 Night 6000 Best call girls ...
VIP Call Girls Pune Sanjana 9907093804 Short 1500 Night 6000 Best call girls ...VIP Call Girls Pune Sanjana 9907093804 Short 1500 Night 6000 Best call girls ...
VIP Call Girls Pune Sanjana 9907093804 Short 1500 Night 6000 Best call girls ...Miss joya
 
Artifacts in Nuclear Medicine with Identifying and resolving artifacts.
Artifacts in Nuclear Medicine with Identifying and resolving artifacts.Artifacts in Nuclear Medicine with Identifying and resolving artifacts.
Artifacts in Nuclear Medicine with Identifying and resolving artifacts.MiadAlsulami
 
Call Girls Cuttack Just Call 9907093804 Top Class Call Girl Service Available
Call Girls Cuttack Just Call 9907093804 Top Class Call Girl Service AvailableCall Girls Cuttack Just Call 9907093804 Top Class Call Girl Service Available
Call Girls Cuttack Just Call 9907093804 Top Class Call Girl Service AvailableDipal Arora
 
CALL ON ➥9907093804 🔝 Call Girls Baramati ( Pune) Girls Service
CALL ON ➥9907093804 🔝 Call Girls Baramati ( Pune)  Girls ServiceCALL ON ➥9907093804 🔝 Call Girls Baramati ( Pune)  Girls Service
CALL ON ➥9907093804 🔝 Call Girls Baramati ( Pune) Girls ServiceMiss joya
 
(Rocky) Jaipur Call Girl - 9521753030 Escorts Service 50% Off with Cash ON De...
(Rocky) Jaipur Call Girl - 9521753030 Escorts Service 50% Off with Cash ON De...(Rocky) Jaipur Call Girl - 9521753030 Escorts Service 50% Off with Cash ON De...
(Rocky) Jaipur Call Girl - 9521753030 Escorts Service 50% Off with Cash ON De...indiancallgirl4rent
 
Call Girls Service Bellary Road Just Call 7001305949 Enjoy College Girls Service
Call Girls Service Bellary Road Just Call 7001305949 Enjoy College Girls ServiceCall Girls Service Bellary Road Just Call 7001305949 Enjoy College Girls Service
Call Girls Service Bellary Road Just Call 7001305949 Enjoy College Girls Servicenarwatsonia7
 
♛VVIP Hyderabad Call Girls Chintalkunta🖕7001035870🖕Riya Kappor Top Call Girl ...
♛VVIP Hyderabad Call Girls Chintalkunta🖕7001035870🖕Riya Kappor Top Call Girl ...♛VVIP Hyderabad Call Girls Chintalkunta🖕7001035870🖕Riya Kappor Top Call Girl ...
♛VVIP Hyderabad Call Girls Chintalkunta🖕7001035870🖕Riya Kappor Top Call Girl ...astropune
 
Best Rate (Hyderabad) Call Girls Jahanuma ⟟ 8250192130 ⟟ High Class Call Girl...
Best Rate (Hyderabad) Call Girls Jahanuma ⟟ 8250192130 ⟟ High Class Call Girl...Best Rate (Hyderabad) Call Girls Jahanuma ⟟ 8250192130 ⟟ High Class Call Girl...
Best Rate (Hyderabad) Call Girls Jahanuma ⟟ 8250192130 ⟟ High Class Call Girl...astropune
 
Call Girls Service Surat Samaira ❤️🍑 8250192130 👄 Independent Escort Service ...
Call Girls Service Surat Samaira ❤️🍑 8250192130 👄 Independent Escort Service ...Call Girls Service Surat Samaira ❤️🍑 8250192130 👄 Independent Escort Service ...
Call Girls Service Surat Samaira ❤️🍑 8250192130 👄 Independent Escort Service ...CALL GIRLS
 
Vip Call Girls Anna Salai Chennai 👉 8250192130 ❣️💯 Top Class Girls Available
Vip Call Girls Anna Salai Chennai 👉 8250192130 ❣️💯 Top Class Girls AvailableVip Call Girls Anna Salai Chennai 👉 8250192130 ❣️💯 Top Class Girls Available
Vip Call Girls Anna Salai Chennai 👉 8250192130 ❣️💯 Top Class Girls AvailableNehru place Escorts
 
Call Girls Service In Shyam Nagar Whatsapp 8445551418 Independent Escort Service
Call Girls Service In Shyam Nagar Whatsapp 8445551418 Independent Escort ServiceCall Girls Service In Shyam Nagar Whatsapp 8445551418 Independent Escort Service
Call Girls Service In Shyam Nagar Whatsapp 8445551418 Independent Escort Serviceparulsinha
 
Call Girls In Andheri East Call 9920874524 Book Hot And Sexy Girls
Call Girls In Andheri East Call 9920874524 Book Hot And Sexy GirlsCall Girls In Andheri East Call 9920874524 Book Hot And Sexy Girls
Call Girls In Andheri East Call 9920874524 Book Hot And Sexy Girlsnehamumbai
 
Russian Call Girls in Pune Tanvi 9907093804 Short 1500 Night 6000 Best call g...
Russian Call Girls in Pune Tanvi 9907093804 Short 1500 Night 6000 Best call g...Russian Call Girls in Pune Tanvi 9907093804 Short 1500 Night 6000 Best call g...
Russian Call Girls in Pune Tanvi 9907093804 Short 1500 Night 6000 Best call g...Miss joya
 
VIP Mumbai Call Girls Hiranandani Gardens Just Call 9920874524 with A/C Room ...
VIP Mumbai Call Girls Hiranandani Gardens Just Call 9920874524 with A/C Room ...VIP Mumbai Call Girls Hiranandani Gardens Just Call 9920874524 with A/C Room ...
VIP Mumbai Call Girls Hiranandani Gardens Just Call 9920874524 with A/C Room ...Garima Khatri
 
Call Girls Horamavu WhatsApp Number 7001035870 Meeting With Bangalore Escorts
Call Girls Horamavu WhatsApp Number 7001035870 Meeting With Bangalore EscortsCall Girls Horamavu WhatsApp Number 7001035870 Meeting With Bangalore Escorts
Call Girls Horamavu WhatsApp Number 7001035870 Meeting With Bangalore Escortsvidya singh
 
Call Girls Service Chennai Jiya 7001305949 Independent Escort Service Chennai
Call Girls Service Chennai Jiya 7001305949 Independent Escort Service ChennaiCall Girls Service Chennai Jiya 7001305949 Independent Escort Service Chennai
Call Girls Service Chennai Jiya 7001305949 Independent Escort Service ChennaiNehru place Escorts
 
High Profile Call Girls Coimbatore Saanvi☎️ 8250192130 Independent Escort Se...
High Profile Call Girls Coimbatore Saanvi☎️  8250192130 Independent Escort Se...High Profile Call Girls Coimbatore Saanvi☎️  8250192130 Independent Escort Se...
High Profile Call Girls Coimbatore Saanvi☎️ 8250192130 Independent Escort Se...narwatsonia7
 
Low Rate Call Girls Pune Esha 9907093804 Short 1500 Night 6000 Best call girl...
Low Rate Call Girls Pune Esha 9907093804 Short 1500 Night 6000 Best call girl...Low Rate Call Girls Pune Esha 9907093804 Short 1500 Night 6000 Best call girl...
Low Rate Call Girls Pune Esha 9907093804 Short 1500 Night 6000 Best call girl...Miss joya
 
Call Girls Service Pune Vaishnavi 9907093804 Short 1500 Night 6000 Best call ...
Call Girls Service Pune Vaishnavi 9907093804 Short 1500 Night 6000 Best call ...Call Girls Service Pune Vaishnavi 9907093804 Short 1500 Night 6000 Best call ...
Call Girls Service Pune Vaishnavi 9907093804 Short 1500 Night 6000 Best call ...Miss joya
 

Recently uploaded (20)

VIP Call Girls Pune Vrinda 9907093804 Short 1500 Night 6000 Best call girls S...
VIP Call Girls Pune Vrinda 9907093804 Short 1500 Night 6000 Best call girls S...VIP Call Girls Pune Vrinda 9907093804 Short 1500 Night 6000 Best call girls S...
VIP Call Girls Pune Vrinda 9907093804 Short 1500 Night 6000 Best call girls S...
 
VIP Call Girls Pune Sanjana 9907093804 Short 1500 Night 6000 Best call girls ...
VIP Call Girls Pune Sanjana 9907093804 Short 1500 Night 6000 Best call girls ...VIP Call Girls Pune Sanjana 9907093804 Short 1500 Night 6000 Best call girls ...
VIP Call Girls Pune Sanjana 9907093804 Short 1500 Night 6000 Best call girls ...
 
Artifacts in Nuclear Medicine with Identifying and resolving artifacts.
Artifacts in Nuclear Medicine with Identifying and resolving artifacts.Artifacts in Nuclear Medicine with Identifying and resolving artifacts.
Artifacts in Nuclear Medicine with Identifying and resolving artifacts.
 
Call Girls Cuttack Just Call 9907093804 Top Class Call Girl Service Available
Call Girls Cuttack Just Call 9907093804 Top Class Call Girl Service AvailableCall Girls Cuttack Just Call 9907093804 Top Class Call Girl Service Available
Call Girls Cuttack Just Call 9907093804 Top Class Call Girl Service Available
 
CALL ON ➥9907093804 🔝 Call Girls Baramati ( Pune) Girls Service
CALL ON ➥9907093804 🔝 Call Girls Baramati ( Pune)  Girls ServiceCALL ON ➥9907093804 🔝 Call Girls Baramati ( Pune)  Girls Service
CALL ON ➥9907093804 🔝 Call Girls Baramati ( Pune) Girls Service
 
(Rocky) Jaipur Call Girl - 9521753030 Escorts Service 50% Off with Cash ON De...
(Rocky) Jaipur Call Girl - 9521753030 Escorts Service 50% Off with Cash ON De...(Rocky) Jaipur Call Girl - 9521753030 Escorts Service 50% Off with Cash ON De...
(Rocky) Jaipur Call Girl - 9521753030 Escorts Service 50% Off with Cash ON De...
 
Call Girls Service Bellary Road Just Call 7001305949 Enjoy College Girls Service
Call Girls Service Bellary Road Just Call 7001305949 Enjoy College Girls ServiceCall Girls Service Bellary Road Just Call 7001305949 Enjoy College Girls Service
Call Girls Service Bellary Road Just Call 7001305949 Enjoy College Girls Service
 
♛VVIP Hyderabad Call Girls Chintalkunta🖕7001035870🖕Riya Kappor Top Call Girl ...
♛VVIP Hyderabad Call Girls Chintalkunta🖕7001035870🖕Riya Kappor Top Call Girl ...♛VVIP Hyderabad Call Girls Chintalkunta🖕7001035870🖕Riya Kappor Top Call Girl ...
♛VVIP Hyderabad Call Girls Chintalkunta🖕7001035870🖕Riya Kappor Top Call Girl ...
 
Best Rate (Hyderabad) Call Girls Jahanuma ⟟ 8250192130 ⟟ High Class Call Girl...
Best Rate (Hyderabad) Call Girls Jahanuma ⟟ 8250192130 ⟟ High Class Call Girl...Best Rate (Hyderabad) Call Girls Jahanuma ⟟ 8250192130 ⟟ High Class Call Girl...
Best Rate (Hyderabad) Call Girls Jahanuma ⟟ 8250192130 ⟟ High Class Call Girl...
 
Call Girls Service Surat Samaira ❤️🍑 8250192130 👄 Independent Escort Service ...
Call Girls Service Surat Samaira ❤️🍑 8250192130 👄 Independent Escort Service ...Call Girls Service Surat Samaira ❤️🍑 8250192130 👄 Independent Escort Service ...
Call Girls Service Surat Samaira ❤️🍑 8250192130 👄 Independent Escort Service ...
 
Vip Call Girls Anna Salai Chennai 👉 8250192130 ❣️💯 Top Class Girls Available
Vip Call Girls Anna Salai Chennai 👉 8250192130 ❣️💯 Top Class Girls AvailableVip Call Girls Anna Salai Chennai 👉 8250192130 ❣️💯 Top Class Girls Available
Vip Call Girls Anna Salai Chennai 👉 8250192130 ❣️💯 Top Class Girls Available
 
Call Girls Service In Shyam Nagar Whatsapp 8445551418 Independent Escort Service
Call Girls Service In Shyam Nagar Whatsapp 8445551418 Independent Escort ServiceCall Girls Service In Shyam Nagar Whatsapp 8445551418 Independent Escort Service
Call Girls Service In Shyam Nagar Whatsapp 8445551418 Independent Escort Service
 
Call Girls In Andheri East Call 9920874524 Book Hot And Sexy Girls
Call Girls In Andheri East Call 9920874524 Book Hot And Sexy GirlsCall Girls In Andheri East Call 9920874524 Book Hot And Sexy Girls
Call Girls In Andheri East Call 9920874524 Book Hot And Sexy Girls
 
Russian Call Girls in Pune Tanvi 9907093804 Short 1500 Night 6000 Best call g...
Russian Call Girls in Pune Tanvi 9907093804 Short 1500 Night 6000 Best call g...Russian Call Girls in Pune Tanvi 9907093804 Short 1500 Night 6000 Best call g...
Russian Call Girls in Pune Tanvi 9907093804 Short 1500 Night 6000 Best call g...
 
VIP Mumbai Call Girls Hiranandani Gardens Just Call 9920874524 with A/C Room ...
VIP Mumbai Call Girls Hiranandani Gardens Just Call 9920874524 with A/C Room ...VIP Mumbai Call Girls Hiranandani Gardens Just Call 9920874524 with A/C Room ...
VIP Mumbai Call Girls Hiranandani Gardens Just Call 9920874524 with A/C Room ...
 
Call Girls Horamavu WhatsApp Number 7001035870 Meeting With Bangalore Escorts
Call Girls Horamavu WhatsApp Number 7001035870 Meeting With Bangalore EscortsCall Girls Horamavu WhatsApp Number 7001035870 Meeting With Bangalore Escorts
Call Girls Horamavu WhatsApp Number 7001035870 Meeting With Bangalore Escorts
 
Call Girls Service Chennai Jiya 7001305949 Independent Escort Service Chennai
Call Girls Service Chennai Jiya 7001305949 Independent Escort Service ChennaiCall Girls Service Chennai Jiya 7001305949 Independent Escort Service Chennai
Call Girls Service Chennai Jiya 7001305949 Independent Escort Service Chennai
 
High Profile Call Girls Coimbatore Saanvi☎️ 8250192130 Independent Escort Se...
High Profile Call Girls Coimbatore Saanvi☎️  8250192130 Independent Escort Se...High Profile Call Girls Coimbatore Saanvi☎️  8250192130 Independent Escort Se...
High Profile Call Girls Coimbatore Saanvi☎️ 8250192130 Independent Escort Se...
 
Low Rate Call Girls Pune Esha 9907093804 Short 1500 Night 6000 Best call girl...
Low Rate Call Girls Pune Esha 9907093804 Short 1500 Night 6000 Best call girl...Low Rate Call Girls Pune Esha 9907093804 Short 1500 Night 6000 Best call girl...
Low Rate Call Girls Pune Esha 9907093804 Short 1500 Night 6000 Best call girl...
 
Call Girls Service Pune Vaishnavi 9907093804 Short 1500 Night 6000 Best call ...
Call Girls Service Pune Vaishnavi 9907093804 Short 1500 Night 6000 Best call ...Call Girls Service Pune Vaishnavi 9907093804 Short 1500 Night 6000 Best call ...
Call Girls Service Pune Vaishnavi 9907093804 Short 1500 Night 6000 Best call ...
 

Pancreatitis sample questions based on neet pg , usmle, plab and fmge pattern (mci screening)